Frage:
Häufige falsche Überzeugungen in der Physik
crskhr
2010-11-18 03:59:59 UTC
view on stackexchange narkive permalink

Nun, in der Mathematik gibt es Dinge, die wahr erscheinen, aber nicht wahr sind. Naive Schüler lassen sich oft von diesen Ergebnissen täuschen.

Lassen Sie mich ein sehr einfaches Beispiel betrachten. Als Kind lernt man diese Formel $$ (a + b) ^ {2} = a ^ {2} + 2 \ cdot a \ cdot b + b ^ {2} $$ Aber als Erwachsener wendet er dieselbe Formel an für Matrizen. Wenn zwei beliebige $ n \ mal n $ Quadratmatrizen gegeben sind, glaubt man, dass dieses Ergebnis wahr ist: $$ (A + B) ^ {2} = A ^ {2} + 2 \ cdot A \ cdot B + B ^ {2} $$ Aber letztendlich ist dies falsch, da Matrizen nicht unbedingt kommutativ sind.

Ich würde gerne wissen, ob solche Dinge auch mit Physikstudenten passieren. Meine Motivation kam von dem folgenden MO-Thread, den viele von Ihnen vielleicht ansehen:

Community-Wiki?
@MArek: Ich habe die Option nicht gefunden. Wenn jemand es kann, ist er willkommen
@Chandru: AFAIK StackExchange hat kürzlich seine diesbezüglichen Regeln geändert, sodass nur Moderatoren ein Fragen-Community-Wiki erstellen können (der Grund dafür ist, dass die CW-Option bei StackOverflow missbraucht wird).
Während ich die Frage interessant finde, denke ich, dass das Beispiel mit Matrizen eher "ein häufiger dummer Fehler" als "ein häufiger falscher Glaube" ist.
@Piotr: Absolut wahr. Aber ich habe viele Schüler gesehen, die dies taten.
Ich denke, MO eignet sich besser für diese Art von Fragen, weil es das Gefühl hat, * nicht alle Konsequenzen von Definitionen richtig verstanden zu haben *. Aber für die Physik hat es einen anderen Geschmack, * die aktuelle physikalische Theorie nicht akzeptieren zu können und / oder die vorherige Theorie loszulassen *.
@Marek Mir scheint, er spricht von populären Missverständnissen, die normalerweise von Autoren und Medienverlegern verbreitet werden und die Situation verschlimmern. Ich bin mir jetzt nicht so sicher, und vielleicht spricht er über Missverständnisse und Fehler, die von Leuten gemacht wurden, die Physik professionell * benutzen *.
Es scheint ein bisschen seltsam, eine einzige Antwort auf eine weiche Frage zu akzeptieren ...
49 antworten:
nibot
2010-11-20 12:37:01 UTC
view on stackexchange narkive permalink

Erstaunlicherweise hat Wikipedia einen Artikel mit dem Titel " Liste häufiger Missverständnisse". Es gibt einen (kurzen) Abschnitt über Physik, in dem Folgendes erwähnt wird:

  • Die Rolle des Coriolis-Effekts in Badewannen und Abflüssen
  • Die Rolle des Drehimpulses für die Fahrradstabilität
  • Der "gleichzeitige" Irrtum bei der Erklärung des Auftriebs, der von einem Tragflügel entwickelt wird
  • Glas ist eigentlich keine hohe Viskosität Flüssigkeit
  • Zusammensetzung der Luft
  • "Blitz schlägt nie zweimal ein"

Der Abschnitt Astronomie enthält auch einige gute :

  • Wenn ein Stern in ein Schwarzes Loch fällt, nimmt seine Anziehungskraft nicht zu.
  • Meteoriten sind nicht wirklich heiß, wenn sie landen. normalerweise sind sie kalt. (Ich würde hinzufügen: Die Erwärmung von Meteoren ist mehr auf die Kompression der Luft vor ihnen zurückzuführen als auf die "Reibung mit der Luft", wie allgemein angenommen wird.)

Einige, die ich würde add:

  • "Sobald sich etwas in der Umlaufbahn befindet, ist es frei von der Schwerkraft der Erde." Sogar gebildete Leute stolpern über diesen einen; Im Internet gibt es viele Leute, die vorschlagen, wir würden die Internationale Raumstation nur in die Mondumlaufbahn "schubsen". Auf einer viel grundlegenderen Ebene des Missverständnisses gibt es die Idee, dass Astronauten "schwerelos" sind, weil sie weit von der Erde entfernt sind.

  • "Es gibt eine Flut auf der Gegenseite der Erde vom Mond / der Sonne, weil die Erde den Ozean vor der Anziehungskraft "schützt". "

Besonders der gleichzeitige Irrtum. Niemand glaubt dir, wenn du ihnen sagst, dass es nicht wahr ist.
Das ist ein großartiger Wikipedia-Artikel!
Das über Flut wäre ein großartiges Fake Science-Poster.
Nur um die 6 physikalischen Beispiele zu verdeutlichen, sind Dinge, die nicht wahr sind (was die Leute glauben) - die 2 astronomischen sind Dinge, die wahr sind (aber die Leute glauben nicht)
Das "Glas ist eine langsame Flüssigkeit" ist nicht wirklich fair - dies leitet sich aus einer alten, erst kürzlich verstandenen Kontroverse darüber ab, ob ein amorpher Glasübergang tatsächlich scharf ist oder ob es sich um einen abgerundeten glatten Nichtphasenübergang handelt . Es war plausibel zu glauben, dass es vor kurzem nicht scharf war, so dass flüssiges und festes Glas nicht durch einen Schmelzübergang getrennt werden.
Der Hauptbeweis für "fließendes Glas" ist, dass alte Fenster unten tendenziell dicker sind. Dies ist jedoch nur ein Artefakt der Herstellungstechnik.
Ich erinnere mich, dass ich im College mit mehreren Leuten (einschließlich meines Lehrers) gestritten habe, dass die Coriolis-Truppe in der durchschnittlichen Badewanne nicht signifikant war, aber ohne Erfolg
Welches ist das als "Zusammensetzung der Luft" beschriebene?
Es wurde aus dem Wikipedia-Artikel gelöscht. Früher hieß es "Luft ist meistens Stickstoff, kein Sauerstoff".
Raskolnikov
2010-11-18 04:44:03 UTC
view on stackexchange narkive permalink

Ein Fehler, auf den ich oft stoße und der so einfach zu machen ist: Menschen glauben irgendwie viszeral, dass schwere Gegenstände schneller fallen als leichte. Abgesehen von den Problemen des Luftwiderstands ist dies offensichtlich falsch, aber es scheint so eingängig zu sein, und ich denke, es hängt irgendwie mit unserem intuitiven Verständnis von Masse als Trägheit zusammen. Da höhere Masse höhere Trägheit bedeutet. Die Leute verstehen das intuitiv, da es mehr Kraft erfordert, einen fetten Kerl zu schieben als einen dünnen. Sie sehen jedoch nicht, dass die Schwerkraft eine der Masse proportionale Kraft ist, so dass mehr Trägheit mit mehr Schwerkraft einhergeht. Infolgedessen die gleiche Gravitationsbeschleunigung für leichte und schwere Körper.

Ich habe sogar den Fehler bemerkt, den professionelle Physiker in umgangssprachlichen Gesprächen gemacht haben.

BEARBEITEN: Ich habe heute diesen Artikel gefunden und ein neues Licht darauf geworfen, warum Missverständnisse in der Physik oder Wissenschaft im Allgemeinen so häufig und so schwer zu beseitigen sind.

Wie bei dem Missverständnis, das ich erwähne, dreht sich Galileo wahrscheinlich in seinem Grab um, um zu hören, dass einige im 21. Jahrhundert dies immer noch glauben! (Es ist jedoch leider wahr.)
+1 für "Luftwiderstand beiseite legen" ... es nervt mich immer, wenn Leute darauf hinweisen, wie die Dinge mit der gleichen Geschwindigkeit fallen, ohne zu erwähnen, dass dies davon ausgeht, dass der Luftwiderstand vernachlässigbar ist. Das Weglassen dieser Klarstellung führt meiner Meinung nach zu mehr Verwirrung, da nur jemand einen Stein neben eine Feder fallen lassen muss, um (fälschlicherweise) zu dem Schluss zu kommen, dass Galileo falsch lag.
Wenn Sie eine feste Erde annehmen, dann sind Sie richtig. Wenn Sie jedoch die Erde bewegen lassen, bewegt sich die Erde schneller, wenn Sie eine größere Masse fallen lassen, als wenn Sie eine kleinere fallen lassen. In diesem Sinne fallen größere Massen schneller ab. Dies hat natürlich nichts mit dem Äquivalenzprinzip zu tun.
Nun, der Teufel steckt immer im Detail.
Es war auch schön zu erwähnen, dass, wenn wir von "gleicher Gravitationsbeschleunigung für leichte und schwere Körper" sprechen, sich der Massenunterschied schließlich in der ** Kraft ** zeigt, mit der die Objekte auf den Boden treffen! (nicht so kontraintuitiv!) :)
@Tim: Ja - deshalb bevorzuge ich wirklich so etwas wie "einen Penny und eine Bowlingkugel" als die Standard "Feder und eine Bowlingkugel".
Es scheint wichtig, hier Galileos berühmtes Gedankenexperiment zu erwähnen. Wenn Sie dem Gedanken folgen, dass schwere Objekte schneller fallen und ein schwer fallendes Objekt in zwei Teile teilen, das große und das große, sollten beide langsamer fallen als das Original. Der obere Teil würde den unteren Teil nach oben ziehen und noch langsamer fahren wollen, daher ein Widerspruch.
Ich denke immer noch gerne, dass große brennende Lastwagen $ 1 + 10 ^ {- 100} $ mal schneller fallen als Federn. [Luftwiderstand vernachlässigen usw.]
Es gibt auch die Komplikation des Auftriebs - ein dichtes Objekt wäre bei gleicher Masse schwerer und würde somit schneller fallen, selbst wenn die Luft reibungsfrei wäre.
@Pavel Sie ignorieren Gezeiteneffekte. Das Aufteilen von etwas in zwei Teile ändert nichts, da die beiden Hälften in unmittelbarer Nähe andere Gezeitenkräfte erfahren als die zwischen zwei weiter voneinander entfernten Objekten.
Erinnert mich an [Rosencrantz und Guildenstern sind tot] (http://www.youtube.com/watch?v=TLKbS4xCmRc&t=1m19s)
Nennen Sie mich einen Idioten, aber ich stimme dem sogenannten "falschen Glauben" an diese Frage immer noch voll und ganz zu. hah aha hha
MatrixFrog
2010-12-03 14:57:08 UTC
view on stackexchange narkive permalink

"Sommer ist, wenn die Erde der Sonne am nächsten ist, und Winter, wenn sie am weitesten entfernt ist."

Es ist wahr, dass die Erdumlaufbahn leicht elliptisch ist, aber die Auswirkung davon, soweit Jahreszeiten ist sehr klein. Zum einen würde dies nicht erklären, warum die Sonne zu verschiedenen Zeiten zu verschiedenen Jahreszeiten aufgeht und untergeht, und wenn dies wahr wäre, würde der gesamte Planet zur gleichen Zeit Sommer haben.

Die Jahreszeiten sind tatsächlich verursacht durch die Neigung der Erde relativ zu ihrer Umlaufbahn um die Sonne.

+1 Obwohl ich das bei 10 Jahren herausgefunden habe
Die Jahreszeit, d. H. Sommer gegen Winter, hängt auch von Ihrer Hemisphäre ab. Auf der Nordhalbkugel, wo die meisten Erdenmenschen leben, ist der Sommer, wenn die Sonne am weitesten entfernt ist, und der Winter, wenn sie am nächsten ist.
Sklivvz
2010-11-20 03:20:52 UTC
view on stackexchange narkive permalink

Ich würde sagen, dass für die meisten Menschen die quadratische Skalierung der kinetischen Energie mit der Geschwindigkeit ein Rätsel ist.

Die Leute verstehen nicht, wie ein Autounfall, wenn Sie doppelt so schnell fahren, tatsächlich viermal so energisch ist, daher die hohe Anzahl rücksichtsloser Fahrer und tödlicher Unfälle.

Wenn Sie aus zwei Metern Höhe springen, treffen Sie mit doppelt so viel kinetischer Energie auf den Boden, nicht mit viermal so viel.
._. Ich muss schlafen ... Energie sparen ... doh. Korrigiert.
Ich erinnere mich, dass mein Fahrer in der High School dies behandelt hat. Mir hat gefallen, dass die Physik mit der Gefahr der Geschwindigkeitsüberschreitung zusammenhängt.
Dies ist eines der mächtigsten Konzepte der Physik, das eine völlig andere Sicht auf die Welt um uns herum eröffnet. Der Term $ v ^ 2 $ ist nur ein Gesicht einer vielseitigen Realität, die auch mit $ W = F x $ oder $ P = v F $ belegt wird. Ich begann darüber in Bezug auf das Brechen von Autos nachzudenken und erkannte, warum es "einfacher" erscheint, mehr G-Kräfte bei niedrigeren Geschwindigkeiten zu erzeugen. Weil es mechanisch ist!
Es ist auch zunächst etwas überraschend zu erfahren, dass kinetische Energie eine vom Messgerät abhängige Größe ist.
Dies wird hier erläutert: http://physics.stackexchange.com/q/535/2451
user68
2010-12-03 18:34:43 UTC
view on stackexchange narkive permalink

Klassische Mechanik ist langweilig und meistens gelöst

... insbesondere bei Strömungsmechanik (-;

)
Ich habe nie jemanden getroffen, der das denken würde. Wenn Sie nichts über Mechanik wissen, können Sie offensichtlich nicht denken, dass es langweilig ist. Und wenn Sie es wissen, sollte Sie nur ein Doppelpendel oder ein Dreikörperproblem davon überzeugen, dass es alles andere als einfach ist. Ich würde Fluiddynamik allerdings nicht als klassische Mechanik bezeichnen. Während es klassisch ist, ist es sicherlich keine Mechanik, sondern eine Feldtheorie. Und das ist der Hauptgrund, warum es schwierig ist.
Ich würde sagen, die meisten Physikstudenten denken, dass dies so ist. Dies hängt eng zusammen mit "alles, was analytisch gelöst werden kann, hat bereits"
Ron Maimon
2011-08-15 11:28:26 UTC
view on stackexchange narkive permalink

Ich werde einige meta-falsche Überzeugungen geben: Dies sind Überzeugungen der Öffentlichkeit, die zufällig wahr sind und die von vielen Physikern mit falschen Korrekturen überkorrigiert werden, basierend auf dem Drang, klug zu erscheinen:

Elektronen bewegen sich langsam einen Draht hinunter

  • Der Glaube: Die Elektronen bewegen sich blitzschnell einen Draht hinunter.

  • die Hyperkorrektur: Im völlig veralteten Drude-Modell bewegen sich Elektronen langsam. In diesem Modell stellen Sie sich vor, dass der Strom von einem klassischen Elektronengas getragen wird, und dividieren den Gesamtstrom durch die Dichte aller elektronischen Ladungen, um die Driftgeschwindigkeit zu erhalten. Dies sagt eine völlig falsche Driftgeschwindigkeit von einigen cm / s voraus, was völliger Unsinn ist, da nur Elektronen in der Nähe der Fermi-Oberfläche zur Leitfähigkeit beitragen. Trotzdem wird diese Hyperkorrektur endlos wiederholt (sie erscheint auch hier).

  • Die beste Antwort: Die elektronischen Wellenfunktionen sind in einem Metall verteilt. Der korrekte Begriff der Elektronengeschwindigkeit ist die Fermi-Geschwindigkeit, die typischerweise enorm ist, da die Wellenlänge etwa 1 Atomradius beträgt. Es ist zwar nicht gleichbedeutend mit der Geschwindigkeit der Elektrizität, die den Draht hinunterfließt (dies ist die Geschwindigkeit der Feldstörungen, ein erheblicher Teil der Lichtgeschwindigkeit), aber sie ist enorm hoch. Verunreinigungen, die Elektronen streuen können, verändern diese Geschwindigkeit, aber nicht so sehr, wie es die naive Hyperkorrektur sagt.

Das Atom ist größtenteils ein leerer Raum

  • Der Glaube: Das Atom ist voller Dinge, deshalb sind Dinge schwer, wenn man dagegen drückt.

  • Die Hyperkorrektur: Im völlig veralteten Rutherford-Bohr-Modell ist das Atom größtenteils ein leerer Raum, das winzige punktförmige Elektron umkreist einen Kern, der den größten Teil der Masse enthält.

  • Die beste Antwort: Aber es ist die Wellenfunktion der Elektronen, die Ihnen sagt, ob etwas ein leerer Raum ist oder nicht. Ein mit elektronischer Wellenfunktion gefüllter Bereich fühlt sich schwer an, da zwei Elektronen nach dem Ausschlussprinzip nicht in den gleichen Raum komprimiert werden können, ohne ihre Wellenfunktion auf sehr hohe räumliche Variationen zu drücken. Atome sind voller elektronischer Wellenfunktionen und daher kein leerer Raum, zumindest nicht nach vernünftiger Definition.

Die Messung in der Quantenmechanik

ist nichts Mystisches
    • Der Glaube: Das Messproblem in der Standardquantenmechanik legt nahe, dass das Bewusstsein irgendwie an Messungen beteiligt ist.

    • Hyperkorrektur: Dekohärenz erklärt all das! Die Quantenmechanik unterscheidet sich in Bezug auf Erleuchtungswerte nicht vom Determinismus.

    • Die beste Antwort: Dekohärenz sagt Ihnen, warum Sie keine Interferenz zwischen klassisch unterschiedlichen Welten oder Geschichten haben und es ist ein wichtiger Teil der Geschichte, wenn die Quantenmechanik genau ist. Aber es sagt Ihnen nicht, warum Sie eine konsistente Geschichte als Welt "wahrnehmen". Sie benötigen ein Wörterbuch zwischen Physik und Wahrnehmung. Da dieses Wörterbuch grundlegend, seltsam und philosophisch ist, ist es wichtig zu erklären, dass dies keine Ausgabe der Physik ist, sondern eine Eingabe, die die mathematische Theorie mit expliziten Sinnesdaten verbindet.

    Es gibt keine Zentrifugalkraft

    • Der Glaube: Wenn sich Dinge drehen, werden sie durch eine Zentrifugalkraft herausgedrückt.
    • Die Hyperkorrektur : Es gibt keine Zentrifugalkraft. Es gibt eine Zentripetalkraft (Zentripetal war ein erfundenes Wort, um die Zentrifugalkraft zu ersetzen), die Sie hineinzieht.
    • Die beste Antwort: Dies trifft offensichtlich aus Sicht des Trägheitsrahmens zu, es gibt keine Zentrifugalkraft, aber wenn Sie es aus der Sicht des rotierenden Objekts betrachten, dann gibt es eine. Es hängt alles von Ihrer Wahl des Referenzrahmens ab.

    Der Urknall geschah überall gleichzeitig

    • Der Glaube: Der Urknall bedeutet, dass das Universum bei begann Ein bestimmter Punkt und wurde von dort aus größer.
    • Die Hyperkorrektur: Der Urknall passierte überall auf einmal, und es ist einfach falsch, sich vorzustellen, dass er an einem einzelnen Punkt in einem erweiterten Modell der Raumzeit passiert . Wenn das Universum offen ist, war der Urknall unendlich groß.
    • Die beste Antwort: Es gibt drei wichtige Vorbehalte: 1. In FRW-Modellen ist der Knallpunkt eine Singularität, also sein Außenraum und Zeit, und es ist unmöglich festzustellen, ob es "wirklich" ein einzelner Punkt oder "wirklich" überall auf einmal ist, also ist es nur eine bedeutungslose Frage. 2. Im Newtonschen "Urknall" -Modell, in dem Sie sich vorstellen, dass das Universum jetzt mit Teilchen gefüllt ist, deren Geschwindigkeit linear proportional zu Ihrer Entfernung von Ihnen entfernt ist, kommt alles aus einem einzigen heraus Punkt! Alle Newtonschen Weltlinien konvergieren auf Ihrer aktuellen Position. Das ist wahr, obwohl das Universum räumlich homogen ist (der Grund, warum es kein Paradox ist, ist, dass galiläische Boosts nicht trivial mit Übersetzungen gemischt werden). 3. Meiner Ansicht nach ist das beste Bild das holographische Bild, bei dem Sie von einem Horizont umgeben sind, der in der Vergangenheit kleiner war. Diese Ansicht ähnelt dem Newtonschen Urknall, da alles aus einer kleinen Region stammt, die von einem dS-kosmologischen Horizont begrenzt wird. Dies ist mathematisch äquivalent zu allem anderen, außer dass das Zeug außerhalb des Horizonts weggeworfen wird, das nicht beobachtet werden kann.

    Ich möchte zugeben, dass ich ein wenig verblüfft war, als mir ein Laie sagte, dass alles in einem Newtonschen Urknall von einem einzigen Ort kommt. Das war völlig uninteressant.

Du solltest unterrichten, Ron.
Schöne Antwort, aber denken Sie an Ihre "beste Antwort" bezüglich der Quantendekohärenz - es könnte klarer sein.
Wenn die Inflation zu unterschiedlichen Zeiten an unterschiedlichen Punkten endet, ist dies dann eine Art Zentrum für unseren Teil des Universums?
@James: Nein, da die verschiedenen Punkte vom Zentrum entfernt in der Vergangenheit liegen. Wenn Sie sich zu diesen Punkten bewegen, bewegen Sie sich in der Zeit zurück und der Horizont zu diesem Zeitpunkt um diesen Punkt in der Zeit zurück / weit im Raum ist immer noch auf den Punkt zentriert. Dies ist ein symmetrischerer Raum als in der Patch-Beschreibung angegeben, weshalb Relativisten Patches nicht mögen, aber sie sind notwendig, um die Quantentheorie zu verstehen.
@Ron Danke, obwohl ich nicht sicher bin, ob ich das wirklich verstanden habe. Kennen Sie eine Referenz, die detaillierter ist?
Ich habe immer geglaubt, dass der Urknall in meiner Kaffeetasse passiert ist. Vielen Dank, dass du mir das weggenommen hast. Ich hatte erwartet, mit dem Verkauf dieser Tasse eine Million Dollar zu verdienen.
xkcd hat das Problem mit der Zentrifugalkraft ziemlich gut behoben: http: //xkcd.com/123/
@Joseph: Ich stimme diesen Comics erst zu, nachdem ich sie erstellt habe, da stimme ich zu. Es ging auch um das "Gummiplatten-GR" -Geschäft, das mich wirklich nervte. Als ich in der High School war, nahm ich das ernst, lernte ein wenig Unterschied, und ich versuchte tatsächlich, die Gummiplatten-Geodäten grob zu lösen, um die Biegung zu bekommen, und ich bekam immer wieder das falsche Vorzeichen, und dann ich erkannte, dass dies alles ein _lie_ ist, die Geodäten auf einer Gummiplatte krümmen sich nicht hinein, dass es nur die Erdgravitation ist, die das Ziehen macht, und sich über diesen "Vorzeichenfehler" Sorgen zu machen, verschwendete viel Zeit.
Marek
2010-11-18 05:13:53 UTC
view on stackexchange narkive permalink

Die Quantenmechanik ist viel zu seltsam, so dass es unmöglich eine korrekte Beschreibung der realen Welt sein kann. Richtig? Ich denke, dazu muss nichts anderes gesagt werden.

Oder vielleicht sind bei einem zweiten Gedanken einige konkretere Überzeugungen und ihre Lösung angebracht:

  1. Die physische Welt muss deterministisch sein (tut es nicht).
  2. Jede mögliche Frage, die Ihnen einfallen kann, muss durch Messung eine genaue Antwort haben (wir beobachten nur, was wir können, nicht was wir wollen to).
  3. Der Zusammenbruch der Wellenfunktion steht im Widerspruch zur endlichen Lichtgeschwindigkeit (es werden keine Informationen übertragen).
Tatsächlich. (Ich muss mehr Zeichen hinzufügen.)
Wahr. Wenn Sie das immer noch stört, sind Sie in guter Gesellschaft (mit Einstein und Bohr?)
@Noldorin: Bohr? "Ja wirklich?" Sie würden keinen strengeren Verfechter der Quantenmechanik finden als Bohr (den ich zu Recht als seinen Vater bezeichnen kann). Einstein dagegen ist eine andere Geschichte. Er konnte seinen Glauben nicht loslassen, dass die Physik * vollständig * sein und * alles * beantworten muss, was wir wissen wollen. Dies führte jedoch zu guten Ergebnissen wie dem EPR-Paradoxon. So kam sein neugieriger Verstand zu einer interessanten Physik, obwohl seine Vorurteile ihn das nicht akzeptieren ließen :-)
@Marek: Deshalb habe ich das Fragezeichen gesetzt, dumm. : P Wer kann Einstein auf jeden Fall die Schuld geben? Ich sicherlich nicht. Ihn als Vorurteil zu bezeichnen, ist nicht nur arrogant, sondern auch äußerst ironisch! Ich möchte diese Debatte nicht fortsetzen, danke.
@Noldorin: warum sollte es arrogant sein? Denken Sie nur daran, dass Einstein selbst die Einbeziehung der kosmologischen Konstante als seinen größten Fehler bezeichnete, und er selbst zugab, dass er zu voreingenommen gegenüber dem stationären Universum war und nicht bereit war, seine Expansion zuzugeben (aber er ließ es schließlich aufgrund von Hubbles experimentellen Beweisen los). Auch Physiker (insbesondere ältere) können Vorurteile haben. Ich sage das nicht mit Verachtung, Einstein war einer der besten Köpfe der Menschheit. Nur dass jeder Vorurteile hat. Obwohl es in diesem Fall ziemlich ironisch ist, weil er bei der Erstellung von QM geholfen hat :-)
Wichtiges Beispiel. Jede neue Theorie (sogar das 1. Gesetz der Newtonschen Dynamik) ist nicht intuitiv, seltsam usw. Was merkwürdig ist, warum ein Jahrhundert nach der Erfindung des QM immer noch Menschen eine solche Einstellung dazu haben. Übrigens: In Shankars Lehrbuch gibt es ein Zitat, das ich liebe: "Für jeden klassischen Physiker oder Philosophen ist das Konzept, dass ein Teilchen gleichzeitig an mehr als einem Ort sein kann, unklar, vage oder, am schlimmsten, tiefgreifend." (Aus dem Speicher, sorry, wenn es einen Fehler gibt.)
@Piotr: Ich denke, dies ist eine Folge der Tatsache, dass wir keine alltäglichen Erfahrungen mit QM haben. In dieser Hinsicht ist QM wirklich nur ein seltsames Tier, das in mikroskopisch kleinen Ecken unseres Universums lauert. ... amüsantes Zitat.
@Marek: Ich denke, es geht nicht nur um mangelnde Erfahrung, sondern auch darum, QM falsch zu lehren (wie oft hört man "Niemand weiß, ob ein elektrisches Feld wirklich existiert oder nur unser Werkzeug, um zu beschreiben, wie sich Elektronen bewegen" oder " Wenn jemand sagt, er verstehe die klassische Wahrscheinlichkeit, muss er lügen! ")
@Piotr: Ich stimme zu, dass es eine Menge Probleme gibt, sich selbst zu unterrichten. Aber darüber möchte ich hier nicht wirklich sprechen. Zum einen ist es nicht zum Thema. Zum anderen würde ich riskieren, mich zu schimpfen und darüber zu schreiben, dass das Schulsystem die schlechteste Institution ist, die jemals geschaffen wurde, und versuchen, jegliche Kreativität bei Kindern zu zerstören und stattdessen einen intensiven Hass und die Angst vor irgendetwas zu wecken, das auch nur annähernd Wissenschaft oder Mathematik ähnelt. Sehen Sie, es ist schon ein Scherz; Also höre ich hier auf.
Es ist Schödinger, der in seinen späteren Jahren ein Verfechter der Kopenhagener Interpretation war. Wohlgemerkt, Einstein und Schrödinger argumentierten nicht gegen die Quantenmechanik als Ganzes, sondern gegen ihre unsinnigen Interpretationen und für eine Vervollständigung der Theorie, die eine vernünftigere erlauben würde. Ich denke darin hatten sie recht. Aber sowohl Einstein als auch Schrödinger waren bereit zu akzeptieren, dass die Theorie nicht deterministisch ist (obwohl Einstein in einer früheren Phase sagte "Gott würfelt nicht", war sein Hauptrindfleisch nicht mit Indeterminismus).
@Marek: Ja, es ist ironisch, da er eine der großen Figuren in der sehr frühen Quantenmechanik war. Trotzdem glaube ich nicht, dass wir auch heute noch sagen können, dass er falsch liegt! Nichts hindert eine andere grundlegendere Theorie daran, die Quantenmechanik zu ersetzen und Einstein Recht zu geben. Auf jeden Fall, fair genug, schlage ich nur vor, dass Sie etwas offener sind. :) (Ich kenne zu viele engstirnige Physiker.)
Sklivvz
2010-11-20 03:28:18 UTC
view on stackexchange narkive permalink

Einige von Weltraum und Science-Fiction abgeleitete Missverständnisse:

  • Ein umlaufender Satellit benötigt Antrieb und diese Umlaufbahn unterscheidet sich vom freien Fall.
  • Sie können tatsächlich a sehen Laserstrahl im freien Raum! Ich habe dies vor einigen Jahren in meinem experimentellen Physikunterricht gesehen, bevor Schlüsselbundlaser üblich waren.
Und das Missverständnis, dass umlaufende Satelliten und andere Raumschiffe mit der vorgetäuschten stationären Kamera kühle Geräusche machen, wenn sie mit 1/1000 ihrer tatsächlichen Geschwindigkeit vorbeifahren.
Ja, aber glaubt das tatsächlich jemand? :-)
Sehen Sie einen Laserstrahl im freien Raum? Sollen nicht alle Photonen direkt in den freien Raum gehen, anstatt dass einige von ihnen den Weg ändern, um in meinen Augen zu landen?
@KimKim Ich habe mich nicht richtig ausgedrückt: Ich habe Leute gesehen, die das glaubten.
Ich mag das in Filmen, in denen die Sicherheitsdienste ein Satellitenbild eines bestimmten Bereichs erhalten müssen und die Leute im Kontrollzentrum einige Tasten drücken und der Satellit sofort seitwärts zum richtigen Ort schwenkt.
Liege ich falsch, dass Satelliten mit Triebwerken ausgestattet sind, um zu manövrieren und ihre Umlaufbahn zu verbessern?
occam98
2010-12-15 07:25:02 UTC
view on stackexchange narkive permalink

Wenn sich ein Objekt bewegt, muss es eine Kraft geben, die es in diese Richtung treibt. Die Schüler denken sehr häufig, dass Kräfte dazu führen, dass Objekte Geschwindigkeit haben, und nicht die Tatsache, dass Kräfte dazu führen, dass Objekte Geschwindigkeit ändern .

In diesem Zusammenhang scheint es viele Physikstudenten auf höchstem Niveau zu verwirren, wenn Beschleunigungs- und Geschwindigkeitsvektoren in entgegengesetzte Richtungen zeigen. Eines meiner Standard-Quiz, als ich Freshman-Physik unterrichtete, bestand darin, einen Ball direkt in der Luft auf und ab zu werfen, dann ein Positions-Zeit-Diagramm zu verteilen und sie zu bitten, die Beschleunigungs- und Geschwindigkeitsvektoren an mehreren wichtigen Punkten zu skizzieren. (Luftwiderstand vernachlässigen). Erhält immer eine interessante Mischung aus Antworten.
Dazu kommt: Wind wird von Bäumen erzeugt, die ihre Blätter herumwedeln :)
Thomas O
2010-12-01 21:37:20 UTC
view on stackexchange narkive permalink

Einige, die ich gehört habe:

  • Schwerere Objekte fallen schneller (dies ist einfach falsch). Größere und kleinere Objekte in einer typischen Erdumgebung würden jedoch aufgrund unterschiedlicher Raten fallen Luftwiderstand, aber die tatsächliche Masse hat keinen Einfluss.

  • Zwei Autos, die mit 60 Meilen pro Stunde kollidieren, sind die gleichen wie ein Auto, das mit 120 Meilen pro Stunde gegen eine Wand kollidiert. Ich denke, MythBusters hat etwas dagegen unternommen.

  • Elektronen bewegen sich sehr schnell auf einer Rennstrecke: Eigentlich bewegen sie sich sehr langsam, aber es ähnelt Newtons Wiege darin, dass es sich um eine kleine Bewegung in einer handelt Ball kann die Energie fast augenblicklich auf den letzten übertragen.

  • Die Gesetze der Thermodynamik wurden von einem Mann in einer Garage mit einigen Magneten gebrochen. Nun, nein, sie scheinen immer noch intakt zu sein, und ein Großteil der modernen Wissenschaft hängt von ihnen ab!

  • Dass sich eine Oszilloskopspur bei teurer Hochgeschwindigkeit schneller als die Lichtgeschwindigkeit bewegt analoge Bereiche (~ 1-2 GHz) Dies ist nicht ganz richtig: Obwohl der Strahl die Oberfläche der CRT schneller als c überstreichen kann (aufgrund der relativ kleinen Bewegung am Hals der CRT), Der Trace kann Informationen nicht schneller als c übertragen.

  • Mehr mit der Chemie zu tun, aber die Tatsache, dass Wasser "Gedächtnis" und all den homöopathischen Unsinn haben kann, der dazu führt, dass H ^ H ^ H ^ H ^ H ^ H ^ H ^ H-Homöopathen sprudeln heraus.

Es ist wichtig, zwischen der Geschwindigkeit der Elektronen zu unterscheiden, aber Elektronen, die sich langsam bewegen, sind nur die halbe Wahrheit. Elektronen haben eine langsame Driftgeschwindigkeit, aber in den meisten Situationen eine hohe thermische Geschwindigkeit.
@Mark Eichenlaub: kann man sagen, dass sie sich schnell bewegen, aber sie reisen nicht schnell. Es ist wahrscheinlich eine sprachbezogene Sache, aber Reisen impliziert eine allgemeine Verschiebung.
Warten Sie, was ist los mit den kollidierenden Autos? Sollte es nicht in allen Referenzrahmen gleich sein? Dies ist sicherlich auch bei einer * elastischen * Kollision der Fall.
@Greg Graviton: Ich empfehle Ihnen, die Folge anzuschauen. Zwei Autos mit 60 Meilen pro Stunde sind ungefähr gleich wie ein Auto gegen eine Wand mit 60 Meilen pro Stunde.
Ah, weil die Energie verwendet wird, um beide Autos zu verformen. Das wäre also wie ein Auto mit 120 Meilen pro Stunde gegen ein anderes Auto anstelle einer Wand. Würde es? Die kinetische Energie hat eine quadratische Geschwindigkeit, aber etwas Seltsames passiert, wenn Sie den Referenzrahmen wechseln.
@Greg: hier ist eine [Analyse des Absturzes] (http://scienceblogs.com/dotphysics/2010/05/mythbusters_energy_explanation.php), die Sie interessieren könnte. Siehe auch [diese Frage] (http://physics.stackexchange.com/q/1368).
Sie sagen, die Objektmasse hat keinen Einfluss auf die Fallgeschwindigkeit in der Luft - auch das ist einfach falsch.
Die Punkte 3 und 5 sind falsch. Elektronen bewegen sich in einem Metall schnell (obwohl nicht so schnell wie das Feld), und es ist durchaus möglich, dass sich der Spurenpunkt eines Oszilloskops schneller als Licht bewegt.
Sudip Paul
2011-01-14 12:41:22 UTC
view on stackexchange narkive permalink

Die Missverständnisse über spezielle Relativitätstheorie und Quantenmechanik sind bekannt. Viele der obigen Beiträge behandeln sie ausführlich. Anstatt dies zu tun, werde ich einige Missverständnisse aus der allgemeinen Physik (z. B. der High School) auflisten:

  1. Wenn ein Körper auf einer Oberfläche ruht, ist die auf ihn einwirkende Kontaktkraft nach oben eine Reaktion darauf sein Gewicht. Dies ist offensichtlich falsch, da Aktion und Reaktion auf verschiedene Körper wirken.
  2. Es gibt viele Missverständnisse über nicht-träge (Pseudo-) Kraft. Mein Physiklehrer hat einmal gesagt, dass nicht träge Kräfte nur dann auftreten, wenn der Körper Kontakt mit einem beschleunigenden Rahmen hat.
  3. Nichts kann sich schneller bewegen als Licht. Stark > Natürlich ist es falsch, wenn Sie nicht den Ausdruck "im Vakuum" hinzufügen. Die Cherenkov-Strahlung tritt auf, wenn sich ein geladenes Teilchen in einem Medium mit einer Geschwindigkeit bewegt, die größer ist als die Lichtgeschwindigkeit in diesem Medium.
  4. Reibung muss immer in die entgegengesetzte Richtung der Gesamtbewegung wirken. Tatsächlich liefert Reibung die notwendige Kraft zum Rollen, ohne die kein Fahrzeug jemals fahren würde. Die richtige Formulierung ist Reibung, die der augenblicklichen Bewegung des Kontaktpunkts entgegenwirkt.
  5. Licht bewegt sich immer in geraden Linien. Auch ohne Gravitationsbiegung, wenn wir einfach ein Medium mit variabler Brechung haben Das Indexlicht folgt einer Kurve. Es ist eine nette Anwendung von Snells Gesetz.
  6. Newtons zweites Gesetz liefert eine Definition von Kraft. Es ist leider ein sehr weit verbreitetes Missverständnis, selbst unter professionellen Physikstudenten. Dies entzieht Newtons zweitem Gesetz jeglichen physischen Inhalt und zwingt ihn (Wortspiel beabsichtigt), eine Tautologie zu werden. Natürlich ist der eigentliche Inhalt des Gesetzes, dass die Kraft durch ein anderes Gesetz (z. B. Gravitation oder Em) gegeben ist und gleich ma ist. Eine überzeugende Diskussion hierzu finden Sie im ersten Band von Feynmans Lectures on Physics. (Es tut mir sehr leid, dass ich das Kapitel oder die Seitenzahl vergessen habe.)
  7. Newtons erstes Gesetz ist vom zweiten Gesetz ableitbar. Der Beweis lautet wie folgt: F = ma. Wenn F = 0, dann ist a = 0, da $ m ~ {} \ neq 0 $ QED. Das Problem ist, dass es ohne das erste Gesetz keine Vorstellung von einem Trägheitsrahmen gibt und die Gesetze sinnlos werden.
  8. In der speziellen Relativitätstheorie wird die Hypothese der Konstanz der Lichtgeschwindigkeit im Vakuum (c) in Bezug auf für alle Beobachter ist überflüssig, weil es aus dem Relativitätsprinzip abgeleitet werden kann. Natürlich kann c variieren, ohne dem Relativitätsprinzip zu widersprechen. Tatsächlich respektiert c in der Newtonschen Mechanik beobachterabhängig das Relativitätsprinzip. Die Konstanz der c-Hypothese ergibt die Lorentz-Transformation, während wir in der Newtonschen Mechanik die gallileische Transformation haben. Wenn Sie immer noch nicht überzeugt sind, schauen Sie sich diese Formulierung der speziellen Relativitätstheorie ohne die zweite Hypothese an. Google doppelt spezielle Relativitätstheorie .
  9. ol>
Zu Punkt 7: So wie ich es kenne, ist die Definition eines Trägheitsrahmens auch im zweiten Gesetz enthalten. Das erste Gesetz ist also buchstäblich ein Sonderfall des zweiten.
Zu Nr. 7: Diese Neuinterpretation des ersten Gesetzes als Definition von Trägheitsrahmen ist heute sehr beliebt, jedoch völlig ohne historische Grundlage. Tatsächlich gibt es in der Newtonschen Mechanik keine logisch realisierbare Definition von Trägheitsrahmen.
Der Punkt eines Laserpointers an einer weit entfernten Wand kann sich schneller bewegen. In Bezug auf Punkt 3 bewegt sich nichts schneller als Licht im Vakuum, dies ist nicht wahr. Phasen einer Lichtwelle in einem Material, virtuelle Teilchenbahnen, die zu einem Pfadintegral beitragen, eine vorgeplante "Welle" der Menge in einem großen Stadion und andere immaterielle Dinge. Es sind materielle Objekte und Informationen werden durch die Lichtgeschwindigkeit eingeschränkt.
Ich glaube auch nicht, dass irgendjemand glaubt, dass 5, 6 und 7 bestenfalls streitig sind, und 8 wird nur von Leuten angegeben, die Maxwells Gleichungen glauben, so dass das Relativitätsprinzip plus die Gültigkeit von Maxwells Gleichungen die Konstanz der Geschwindigkeit von impliziert Licht.
Omega Centauri
2010-12-03 00:01:55 UTC
view on stackexchange narkive permalink

Wenn Sie es irgendwie schaffen, ein Gesetz der Physik zu brechen, wird das Universum verschwinden!

Ich mag das. Und ich denke, ich werde etwas hinzufügen, das damit zusammenhängt.
Glücklicherweise wird es dann durch eine Sicherungskopie ersetzt
Woher wissen wir, dass es nicht verschwinden wird?
Noldorin
2010-11-18 04:33:39 UTC
view on stackexchange narkive permalink

Historisch gesehen wurde das Konzept der absoluten Geschwindigkeit bis zur Zeit von Galileo Galilei im frühen 17. Jahrhundert allgemein angenommen. Als naives Kind ist es schon vor dem Studium der Grundlagenphysik überraschend einfach, daran zu glauben!

Die Idee der absoluten Geschwindigkeit besagt, dass alle Geschwindigkeiten in Bezug auf einen absoluten Bezugsrahmen festgelegt sind. Galileo hat gezeigt, dass die Geschwindigkeit relativ zu Ihrem Bezugsrahmen ist, ein Prinzip, das als galiläische Relativitätstheorie bekannt ist. Dies wurde später von Isaac Newton vollständig quantifiziert, der auch vorschlug, dass die Beschleunigung in Bezug auf Trägheitsrahmen unveränderlich ist.

Ähm, glaubte Newton nicht fest an einen absoluten Bezugsrahmen, den Äther? Und glaubten Wissenschaftler nicht allgemein, dass der Äther für Maxwells Gleichungen erforderlich war, bis eine spezielle Relativitätstheorie eintrat?
Der absolute Bezugsrahmen schließt die galiläische Relativitätstheorie überhaupt nicht aus!
Mark Eichenlaub
2010-11-20 02:57:22 UTC
view on stackexchange narkive permalink

Als Tutor habe ich häufig Gespräche wie diese:

"Wir haben also herausgefunden, dass wenn ich meinen Stift mit 2 m / s hochwerfe, er 20 cm hoch wird. Wie hoch wird er gehen Wenn ich es mit 4 m / s hochwerfe? "

" 40 cm. "

" Nun, okay, lassen Sie uns das noch einmal durcharbeiten ... "

[Wir finden heraus, dass die Antwort 80 cm beträgt]

"Wenn ich es also doppelt so schnell hochwerfe, geht es viermal so hoch, weil es doppelt so lange dauert, bis ich zum komme top, geht aber auch doppelt so schnell. "

" Okay. "

" Was ist, wenn ich es dreimal so schnell hochwerfe? Wie oft so hoch wird es gehen ? "

" Sechs. "

Dies ist kein Missverständnis über kinetische Energie, sondern ein Mangel an Verständnis dafür, was Skalierung ist. Wenn Studenten dieses Konzept vermissen, ist fast die gesamte Physik schwieriger zu diskutieren.

In dem Beispiel klingt es aus den gegebenen Informationen wie ein Problem, einfach nicht zu verstehen, dass der Begriff quadratisch und nicht proportional ist.
@Mark Nein, nicht wirklich. Sie "wissen", dass der Begriff quadratisch ist. Sie können sofort die Formel KE = 1 / 2mv ^ 2 rezitieren. Aber sie verstehen nicht wirklich, was das bedeutet. Sie denken, dass die einzige Möglichkeit, Fragen zu beantworten, was passieren würde, wenn wir das Ding doppelt so schnell werfen, darin besteht, einige Zahlen in die Formel einzufügen. Die Idee, die Exponenten in Formeln zu betrachten und daraus physikalische Einsichten zu gewinnen, ist ihnen fremd. Es ist für Sie selbstverständlich, weil Sie die Grundlagen der Physik und Mathematik recht gut kennen, aber für die Schüler ist es ein seltsamer und ungewöhnlicher Trick. Es ist leicht zu vergessen, wie wenig Sie vor langer Zeit wussten.
Okay, jetzt kommen wir dem Problem auf den Grund. Solche Schüler brauchen also Hilfe, um den Unterschied zwischen Multiplizieren mit einer Konstanten und Quadrieren zu verstehen! (Was ich sagen wollte, aber wir könnten dies den ganzen Tag zu beiden Seiten diskutieren.) Ich nehme an, Sie müssten den gesamten Prozess mit (jedem?) Schüler durchlaufen und alle Probleme entdecken und beheben.
Ja genau. Die Intuition über Mathematik aufzubauen ist ein langsamer Prozess, aber es ist interessant zu beobachten, wie ein Schüler im Laufe eines Jahres Fortschritte macht.
Das Problem ist nicht, dass sie die * Physik * nicht extrahieren; Es ist so, dass sie die ** Mathematik ** nicht extrahieren. Sie haben jahrelang Linearität in ihr Gehirn geschlagen, während sie sich kaum darum kümmerten; und obwohl sie andere Funktionen berechnen können, haben sie nie eine Intuition darüber entwickelt. Sie erwarten also unbewusst, dass alles linear wirkt - und weil sie die Mathematik nicht aus der Formel für kinetische Energie * extrahieren *, glauben sie nicht, dass sie das Quadrat von irgendetwas berechnen möchten.
Marek
2010-12-03 15:48:37 UTC
view on stackexchange narkive permalink

Sie benötigen etwas mehr, um mehr zu erhalten.

Hier geht es um neu auftretende makroskopische Eigenschaften mikroskopischer Gesetze. Einige Leute können nicht verstehen, dass Statistiken mächtig genug sind, um einen scheinbar zufälligen Haufen von Molekülen plötzlich makroskopische Eigenschaften wie fest oder magnetisch zeigen zu lassen, und sie denken, dass es eine Hand Gottes ist erforderlich, um dies zu erreichen.

Das beste Beispiel für einen Widerspruch zu diesem Prinzip sind lebende Organismen. Sie bestehen aus nichts anderem als wenigen physikalischen Gesetzen und einer großen Anzahl von Molekülen. Alles, was benötigt wurde, waren Statistiken und natürliche Selektion.


Ich stimme jedoch zu, dass es ziemlich erstaunlich ist, dass unsere gesamte Natur spontan nur aus Partikeln hervorgehen kann, wenn genügend Raum und Zeit vorhanden sind.

Es ist keineswegs offensichtlich, dass Statistiken und natürliche Selektion Lebewesen hervorbringen können. Die Theorie ist noch nicht auf quantitativer Ebene. Wie würden Sie die Größenordnung der Zeit für die Evolution schätzen? Darwin konnte dies nur phänomenologisch tun, und zwar mit den bekannten Raten, mit denen die Domestizierung die Tiermerkmale verändert. Wenn Sie versuchen, die Änderungsrate mithilfe moderner Genetik naiv abzuschätzen, erhalten Sie eine Schätzung, die unglaublich lang ist, wie Pauli vor vielen Jahren bemerkt hat. In diesem Fall weist der Volksglaube darauf hin, dass wir die Evolution nicht quantitativ verstehen.
@Ron: Ich sage nicht, dass es offensichtlich ist, nur dass es möglich ist (d. H. Sie brauchen Gott nicht, um Leben zu schaffen). Und es ist sicherlich die einzige wissenschaftliche Erklärung, die wir haben. Was also getan werden muss, ist "nur" eine bessere Quantifizierung relevanter Prozesse.
Ich sage, obwohl Darwins ursprüngliche Idee nahezu genau ist, kann man komplexe Evolutionssysteme nicht auf einfache mechanische Weise beschreiben, wie dies die moderne Syntheseentwicklung tut. Das Einbeziehen komplexer Systemeffekte ist in diesem Fall kaum von intelligentem Design zu unterscheiden, da ein großes Computersystem dieser Art in einem sinnvollen Sinne intelligent ist, um auf Systemebene kohärente logische Mutationen und intelligente Auswahl erzeugen zu können. Dies ist Behe ​​näher als der modernen Synthese.
Pete
2010-12-03 22:20:47 UTC
view on stackexchange narkive permalink

Die häufigsten Missverständnisse betreffen Schwerkraft :

(1) Die Schwerkraft wird in der Umlaufbahn des Space Shuttles ausgeschaltet, da die Astronauten schwerelos sind

Die Schwerkraft liegt im Vergleich zur Erdoberfläche bei etwa 80%. Die Astronauten sind schwerelos, weil sich das Shuttle im freien Fall (Orbit) befindet. Wenn es keine Schwerkraft gäbe, könnte das Shuttle nicht umkreisen.

(2) Die Schwerkraft wird von der sich drehenden Erde erzeugt. Wenn sich die Erde nicht mehr dreht, wird die Schwerkraft ausgeschaltet.

Die Schwerkraft wird aufgrund der Erdmasse und der Masse des Objekts erzeugt. die beiden üben einen gegenseitigen Zug aus. Es gibt einige kleinere Effekte, die mit der sich drehenden Erde verbunden sind (z. B. der Coriolis-Effekt), aber die Schwerkraft würde immer noch gut funktionieren, wenn sich die Erde nicht mehr dreht.

Es gibt ein weiteres großes Missverständnis über die Aufprallkraft zwischen einem LKW und ein kleines Auto bei einer Kollision:

(3) Der LKW übt eine größere Aufprallkraft auf das Auto aus als das Auto auf dem LKW.

Während Der Schaden kann sicherlich ungleich sein, die Kräfte sind.

(3) ist nicht wirklich ein Missverständnis, wenn * Gewalt * umgangssprachlich verstanden wird (normale Menschen verwenden nicht wirklich Wörter wie Gewalt und arbeiten im physischen Sinne). Insbesondere der Lkw wird viel mehr Schwung haben, so dass die Situation sicherlich asymmetrisch ist. Aber sonst mag ich diese. +1
Cristi Stoica
2010-12-05 01:18:37 UTC
view on stackexchange narkive permalink

Ich höre von Zeit zu Zeit Leute mit hoher Ausbildung und Fachkenntnissen in Physik (im Gegensatz zu denen, die glauben, dass schwere Objekte schneller fallen als leichte), die folgende Behauptung aufstellen:

... Quantenmechanik zeigt an dass bestimmte physikalische Größen nur einen zählbaren Satz diskreter Werte annehmen können. Infolgedessen befürworten viele aktuelle Ansätze zu grundlegenden Fragen in der Physik und Kosmologie neuartige diskrete oder "digitale" Naturbilder.

( Essay-Wettbewerb "Is Reality Digital or Analog" bei FQXi)

Die diskreten Spektren einiger Quantenobservablen implizieren nicht, dass die Natur, insbesondere die Raumzeit, grundsätzlich diskret ist. Das Spektrum eines kontinuierlichen Operators, der auf Hilbert-Räume wirkt [der ein topologischer (Vektor-) Raum ist und daher kontinuierlich ist], hat oft einen diskreten Teil. Dies hat nichts damit zu tun, dass die Raumzeit diskret ist. Wenn es sich (irgendwann) als diskret herausstellt, dann aus anderen Gründen.

Die genaue Aussage ist, dass die Quantenmechanik eine fundamentale Diskretion des Zustandsraums, des Positions-Impuls-Phasenraums in Vielfachen der Planckschen Konstante h impliziert. Die positivistischen Argumente, die eine Grobkörnung im Phasenraum implizieren, implizieren auch eine Grobkörnung des physikalischen Raums auf der Planck-Länge, und dies wird durch Theorien der Quantengravitation geboren. Das ist also kein Mythos. Der Mythos besagt nur, dass der richtige Weg zur Diskretisierung darin besteht, die Raumzeit durch ein endliches Gitter zu ersetzen.
Henry
2010-12-06 03:45:13 UTC
view on stackexchange narkive permalink

Wenn Sie Fahrrad fahren und das Vorderrad nach links drehen, lenkt das Fahrrad nach links.

http://en.wikipedia.org/wiki/Countersteering
Pavel Radzivilovsky
2010-12-01 03:40:49 UTC
view on stackexchange narkive permalink

"Das Verbrennen von Kohle zum Heizen ist aufgrund thermodynamischer Verluste im Kohlekraftwerk effizienter als elektrisches"

Dies ist falsch, obwohl es wahr ist, dass die Umwandlung von elektrischer Energie in Hitze wäre noch schlimmer. Der richtige Weg, um mit einer bestimmten Menge an Wärmequelle zu heizen, ist folgender:

  • Verbrennen bei hoher Temperatur
  • Arbeiten mit einer Wärmemaschine zwischen T hoch sub> und die Umgebung
  • nutzen die Arbeit, um eine Klimaanlage mit Strom zu versorgen, um Wärme aus der T -Umgebung sub> in den T -Raum sub> zu bringen.
  • Dies ergibt einen Wirkungsgrad von mehr als 1 (mehr Wärme, die in den Raum gebracht wird als durch Verbrennen erzeugte Wärme) und eine Nettokühlung der Umgebung.

    http://physics.stackexchange.com/questions/1493/how-efficient-is-an-electric-heater
    Vielen Dank für verwandte Fragen, aber es wird dieses Thema nicht wirklich diskutiert.
    In der Praxis trifft dies jedoch zu: Die Umwandlung der Kohle in Elektrizität und der Betrieb einer Wärmepumpe sind in der Praxis weniger effizient als die direkte Verbrennung der Kohle. Ich glaube nicht, dass man eine praktische Faustregel als Trugschluss bezeichnen kann.
    Marek
    2010-12-03 15:41:40 UTC
    view on stackexchange narkive permalink

    Warum muss ich dieses Gesetz lernen, wenn sie es alle paar Jahre ändern?

    Dies hat mit der Tatsache zu tun, dass einige (tatsächlich viele) Menschen Ich glaube, dass der Fortschritt in der Physik in Form einer Revolution erfolgt und (insbesondere) dass wir eines Tages Gesetze finden könnten, die allem widersprechen, was wir bis dahin wussten.

    Nun, Wenn man sich die Geschichte der Physik genau ansieht, sollte klar werden, dass Fortschritt immer nur evolutionär war. Selbst wenn eine Idee eine Revolution in der Art und Weise brauchte, wie Menschen denken (wie bei SR und QM), stellte sich heraus, dass dies immer nur eine Verallgemeinerung unserer vorherigen Ideen war (also haben sowohl SR als auch QM schöne klassische Grenzen, die fallen mit der Newtonschen Mechanik zusammen.

    Abgesehen von den nutzlosen philosophischen Ansichten (wie wir vielleicht in Matrix leben oder nicht wissen, ob die Sonne morgen sicher aufgehen wird) ist es ziemlich sicher, dass unser Universum ein verständlicher Ort ist und unsere Theorien sind immer bessere Annäherungen an die Realität. Es wird also immer nützlich sein, die Newtonsche Mechanik zu lernen, auch in einer Million Jahren.

    student
    2010-12-01 04:17:44 UTC
    view on stackexchange narkive permalink

    Ich denke, eine verbreitete falsche Annahme ist, dass sich eine Lichtmühle dreht, weil Photonen auf der glänzenden Seite (wo sie reflektiert werden) mehr Impuls abgeben als auf der schwarzen Seite (wo sie absorbiert werden).

    Ich finde es ziemlich erstaunlich zu sehen, dass viele Leute dies glauben, obwohl sich eine Lichtmühle in die entgegengesetzte Richtung dreht, als durch diese Erklärung vorhergesagt.

    Es hängt von dem Vakuum ab, das Sie haben. Wenn das Vakuum nicht hoch genug ist, wird die schwarze Seite stärker erwärmt als die glänzende Seite. Aufgrund der Luftkonvektion und einiger Kanteneffekte kommt es zu einer Bewegung von der schwarzen Seite zur glänzenden Seite. Wenn das Vakuum hoch genug ist, bewegt der Strahlungsdruck die Mühle. Es kommt also auf die Lichtmühle an.
    Jemand sollte eine leichte Mühle erfinden, in der der Druck im Inneren so verändert werden kann, dass sie in beide Richtungen funktioniert. Es würde Spaß machen, dies Studenten und Nichtwissenschaftlern zu zeigen und sie zu bitten, dies zu erklären.
    Ron Maimon
    2011-08-15 12:23:24 UTC
    view on stackexchange narkive permalink

    Hier ist eine weitere Liste falscher Überzeugungen. Diese werden von Wissenschafts-Popularisierern gehalten. Ob sie diesen Überzeugungen tatsächlich glauben oder sie nur aussprechen, um mehr Zuschauer zu gewinnen, ist eine unbeantwortbare Frage:

    Der gekrümmte Raum in der Nähe eines massiven Objekts kann als deformierte Gummiplatte

    Dieser ist leider Einstein zu verdanken. Sie legen Bälle auf eine Gummiplatte und sehen, dass sie aufeinander zu rollen. Der Grund, warum dies eine schreckliche Erklärung ist, ist, dass die Schwerkraft der Erde das Ziehen übernimmt, nicht der gekrümmte Raum. Die tatsächliche Geodäten auf einem gekrümmten Raum wie der Gummiplatte werden von der zentralen Masse abgestoßen. Der Grund, warum sich die Dinge in der Relativitätstheorie anziehen, ist der Zeitdilatationsfaktor, und dies ist der dominierende Effekt. Es ist genauso einfach, die Dinge richtig zu erklären, wenn sich die Zeit in der Nähe eines massiven Objekts verlangsamt und die Weltlinien versuchen, ihre richtige Zeit mit bestimmten festen Endpunkten zu maximieren, aber Popularisierer tun dies niemals.

    A. Eine variable Lichtgeschwindigkeit kann die Inflation ersetzen.

    Dies erschien kürzlich in einer populären Show und basiert auf der folgenden falschen Idee: Wenn sich das Licht zu frühen Zeiten schneller bewegte, könnte das gesamte Universum in Kommunikation gewesen sein ! Der Grund dafür ist, dass man, unabhängig davon, wie sich die Lichtgeschwindigkeit ändert, die Raumzeit in Bezug auf die Schnittpunkte von Lichtkegeln neu koordinieren kann. Wenn sich diese Lichtkegel nicht teilen, sondern verschmelzen, erhalten Sie dasselbe Kommunikationsparadoxon. - Neue Regionen, die in kausalen Kontakt kommen, kommen zum ersten Mal in kausalen Kontakt.

    Mesonen und Baryonen bestehen aus Quarks wie Atome aus Protonen, Neutronen und Elektronen.

    Das ist heimtückisch, weil es für schwere Mesonen gilt. Aber es ist viel mehr falsch als wahr für Pionen und Protonen und alle Anregungen bei weniger als 1GeV aufgrund der Vakuumkondensate. Es gibt kein vernünftiges Modell für leichte Pionen, das ihre Goldstone-Natur nicht berücksichtigt. Diese Art der Erklärung lässt auch Nambu und Skyrme aus, die beide zu lange zu Unrecht ignoriert wurden.

    Stringtheorie ist eine Theorie der Strings

    Dieses Bild ist nicht gut für jemanden, der hat noch keinen Sinn für Stringtheorie, denn wenn Sie anfangen, selbstgemachte Modelle relativistischer Strings zu erstellen, werden Sie nie so etwas wie die richtige Stringtheorie bekommen. Die Saiten, die Sie sich naiv vorstellen, haben nicht die speziellen Lichtkegel-Wechselwirkungen, die die Saiten im Mandelstam-Bild haben, und sie würden der Dualität von Dolen Horn Schmidt nicht gehorchen. Es wären nur Konglomerate von Punktpartikeln, die durch Gummibänder zusammengehalten werden. Sie hätten das falsche Spektrum und wären voller Geister.

    Der einzig richtige Weg zu sagen, was Strings sind, besteht darin, sofort zu sagen, dass es sich um S-Matrix-Zustände handelt und dass dies der Fall ist Entwickelt als S-Matrix-Theorie mit linearen Regge-Trajektorien. Sie haben ein String-Bild, aber die Einschränkung, dass der Austausch von Strings im S-Kanal doppelt so groß ist wie der Austausch im T-Kanal, ist ebenso wichtig wie historisch gesehen. Ohne dies sind Sie selbst mit der Nambu-Aktion nicht in der Lage, Interaktionen zu integrieren. Es ist nicht offensichtlich, dass die Wechselwirkungen topologisch sind, es sei denn, Sie kennen Dolan Horn Schmidt.

    Es ist auch wichtig zu erkennen, dass String-Interaktionen etwas ganzheitlich sind (dass sie auf dem Lichtkegel lokal werden, ist die Überraschung, nicht umgekehrt). Sie fügen sie in der Störungstheorie Reihenfolge für Reihenfolge hinzu, indem Sie Einheitlichkeit fordern und nicht fragen, was passiert, wenn zwei Zeichenfolgen im üblichen Sinne kollidieren. Diese "Saiten" sind seltsame neue Dinge, die aus den Kau-Ismen der 1960er Jahre hervorgegangen sind, und ihre engsten Verwandten sind Flusslinien in der Eichentheorie oder Fischnetz-Feynman-Diagramme, keine Ansammlung von Punktmassen, die durch federartige Kräfte zusammengehalten werden.

    Es gibt mehr als die Chance eines Schneeballs Hölle für große zusätzliche Dimensionen

    Die Idee, dass es große zusätzliche Dimensionen gibt, war im Jahr 2000 sehr beliebt, ist aber völlig absurd. Große zusätzliche Abmessungen reduzieren die Planck-Masse auf etwa einen TeV, wodurch Neutrinos generische Majorana-Massen erhalten, die im KeV-MeV-Bereich liegen. Sie müssen also eine Feinabstimmung vornehmen. Sie führen zu einem im Wesentlichen sofortigen Protonenzerfall und zu großen CP-Verstößen bei starken Wechselwirkungen. Sie müssen daher weitere Feinabstimmungen vornehmen. Um den Protonenzerfall zu vermeiden, gibt es einen cleveren Mechanismus von Arkani-Hamed und Schmalz, der die Quarks und Leptonen an verschiedenen Stellen in den zusätzlichen Dimensionen platziert. Diese Idee ist nur auf den ersten Blick ansprechend, da SU (2) und U (1) des Standardmodells in den zusätzlichen Abmessungen erweitert werden müssen, was sich sofort auf deren Lauf auswirkt. Die Theorie sagt eindeutig und modellunabhängig voraus, dass die Unterdrückung des Protonenzerfalls eine enorme Elektroschwäche erfordert, die um einen TeV herum läuft. Das ist ein Signal, von dem Sie bei 100GeV-Kollisionen keinen Hinweis gesehen haben. Komm schon. Wie stabilisieren Sie außerdem große Dimensionen? Es ist die gleiche Feinabstimmung wie zuvor, daher ist die Anzahl der Probleme gestiegen.

    Eine niedrige Planck-Skala würde die Vorhersagbarkeit der Stringtheorie vollständig zerstören. Sie können eine Menge Dinge in große Dimensionen drücken. Meiner Meinung nach ist es diese Marke der Stringtheorie, die die Kritiker zu Recht als grundsätzlich nicht vorhersagbar kritisieren.

    Was meinst du mit "Geodäten stoßen sich gegenseitig ab"? Meine Vorstellung von einer Geodät ist, dass es einfach eine Verallgemeinerung einer geraden Linie ist.
    @CHM: Ich meinte, dass sich die Geodäten von der Zentralmasse weg krümmen, so dass sich zwei Partikel, die die Gummiplatte wie in Popularisierungen dargestellt verformen und sich jeweils auf einer Geodät bewegen, gegenseitig abstoßen. Die Bewegung von Objekten, die durch die Schwerkraft auf eine Gummiplatte gezogen werden, ist keine geodätische.
    Marek
    2010-11-18 05:10:03 UTC
    view on stackexchange narkive permalink

    Eine weit verbreitete Überzeugung (ich denke aufgrund populärer Bücher wie Hawkings) ist, dass GTR niemals, niemals quantisiert werden kann und man immer Unendlichkeiten und bla bla erhält. Nun, es kann in vielen Situationen und in vielen Theorien. Was eigentlich gemeint ist, ist, dass GTR keine renormalisierbare Quantenfeldtheorie auf naive Weise ist. Auf diese Spezifikation wird jedoch nie ausdrücklich hingewiesen, so dass die Menschen den falschen Eindruck bekommen, dass die Quantengravitation etwas ist, das völlig außerhalb des Bereichs der gegenwärtigen Physik liegt. Nun, Überraschung, Überraschung, ist es nicht. Wir können viele Gravitationseffekte (wie Wellen) quantifizieren, wir verstehen, dass Schwarze Löcher Entropie haben, wir verstehen, dass sie Hawking-Strahlung erzeugen und schließlich verschwinden usw. Und wenn wir davon ausgehen, dass die Stringtheorie korrekt ist, können wir viel mehr darüber vorhersagen.

    Die Quantisierung von Gravitationswellen in einem flachen Hintergrund oder QFT in schwach gekrümmter Raumzeit ist etwas ganz anderes als die Quantengravitation.
    @Vagelford: sicher. Ich hatte nicht die Absicht, einen vollständigen Bericht über das zu geben, was über die Quantengravitation bekannt ist (hauptsächlich, weil ich nicht dazu berechtigt bin). Nur dass es möglich ist, zumindest etwas zu tun (im Gegensatz zu dem, was die Leute normalerweise denken). Zum Beispiel habe ich kürzlich irgendwo gelesen, dass niemand weiß, wie Sterne funktionieren, weil GR und Kernphysik niemals zusammen behandelt werden können. Unglaublich, oder?
    OK. Diese Aussage über Sterne ist einfach albern.
    Pavel Radzivilovsky
    2010-12-01 15:50:14 UTC
    view on stackexchange narkive permalink

    Einige Sportlehrer würden Ihnen sagen:

    Das Laufen auf einem Laufband ist einfacher, weil Sie nur springen müssen, während Sie auf der Straße auch vorwärts schieben

    Das ist vielleicht nicht der ganze Unterschied. Wenn Ihr Fuß auf das Laufband stößt, kann dies den Gurt etwas verlangsamen. Dann kann der Gurt die Geschwindigkeit wieder erhöhen, während Sie in der Luft sind. Daher ist die Geschwindigkeit, während Ihr Fuß in Kontakt ist, möglicherweise langsamer als die Durchschnittsgeschwindigkeit. Wenn Sie versuchen, ein ausgeschaltetes Laufband zu nehmen, werden Sie feststellen, dass Sie durch Halten der Schienen darauf laufen können, indem Sie den Gurt mit den Füßen nach hinten drücken. Empirisch bemerkte ich, dass meine Herzfrequenz und meine wahrgenommene Anstrengung auf einem Laufband nicht so hoch sind wie auf einer Strecke mit der gleichen Geschwindigkeit.
    Nun, es hängt wahrscheinlich vom Laufband ab. Es könnte aufgrund eines von Ihnen beschriebenen Konstruktionsfehlers einfacher sein oder auch nicht, aber zurück zum Thema der Frage: Galileo-Transformationen funktionieren :)
    Das Laufen auf einem Laufband in der geneigten Einstellung sollte nicht mehr Arbeit leisten als das Laufen auf einem "flachen" Laufband, aber alle haben Bergmodi
    Carl Brannen
    2011-01-28 21:46:02 UTC
    view on stackexchange narkive permalink

    Als Ausbilder habe ich große Schwierigkeiten, Newtons 3. Gesetz zu lehren: "Für jede Handlung gibt es eine entgegengesetzte und gleiche Reaktion." Dies ist eine sehr grundlegende und sehr alte Physik, aber es ist schwer zu lehren. Ein typisches Beispiel für die falsche Antwort der Reaktionskraft ist das Folgende:

    Ich halte einen Apfel in der Hand. Die Erde zieht mit einer Gravitationskraft auf den Apfel. Was ist die Reaktionskraft darauf?

    Die Antwort, die die meisten von ihnen geben werden, ist, dass die Reaktionskraft "meine Hand, die auf den Apfel drückt" ist. Arggghhhh! Natürlich ist die Reaktionskraft "der Apfel, der auf der Erde hochzieht".

    Die Schüler erkennen nicht, dass die entgegengesetzten und gleichen Reaktionen zwischen demselben Objektpaar stattfinden müssen. Das heißt, Kräfte entstehen paarweise. Ich wünschte, sie würden das Gesetz einfach umbenennen, damit klarer wird, dass die Reaktionskraft zwischen demselben Objektpaar wirken muss.

    Ich demonstriere das Gesetz, indem ich eine lange Feder in meinen Händen halte und ihnen zu sagen, dass Kräfte wie dieser Frühling sind. Wenn es eine Kraft auf ein Ende ausübt, übt es eine Kraft auf das andere aus (nehmen Sie eine masselose Feder an). Nächstes Quartal werde ich einige extremere Maßnahmen ergreifen, offensichtlich scheitere ich.

    Cem
    2010-11-18 05:05:38 UTC
    view on stackexchange narkive permalink

    Während die anderen Antworten absolut richtig sind, sind sie in ihrer Darstellung sehr subtil. Ein sehr großer falscher Glaube in der Elektrowissenschaft ist jedoch, dass Strom im Gleichstrom vom + -Anschluss zum - -Anschluss "fließt".

    Dabei spielt es keine Rolle, welchen Weg Sie wählen Da es sich um einen Strom mit nahezu Lichtgeschwindigkeit handelt, bewegen sich die Elektronen tatsächlich von - Anschluss zu + Anschluss (verstehen Sie mich nicht falsch). Elektronen bewegen sich beim Driften NICHT in die Nähe der Lichtgeschwindigkeit. Sie bewegen sich in der Größenordnung von Zentimetern pro Sekunde.) . Daher bewegt sich der Strom tatsächlich von - nach +.

    Und es wäre interessant festzustellen, dass keine Seele im professionellen Bereich der Elektrotechnik den Strom von - nach + betrachtet, da dies zu Inkonsistenzen mit seinem führen würde / ihre Kollegen.

    Wenn Strom immer als Elektronen von - nach + fließt, erklären Sie bitte, wie der Hall-Koeffizient in vielen Materialien, wie z. B. p-Halbleitern, positiv sein kann.
    Ich hätte mich entschuldigen müssen, ich hätte nur eine einfache, lineare, leitende RCL-Schaltung in Betracht ziehen sollen. Es gibt Beispiele für Strom, der in umgekehrter Richtung fließt. Die allgemeine Regel eines "imaginären" Stroms, der umgekehrt zur Richtung fließender Elektronen fließt, ist jedoch immer noch eine Tatsache, eine akzeptierte Tatsache, die völlig willkürlich ist und bei der der Fluss keine Berechnungsfehler verursachen würde wurde als - bis + genommen.
    Die beste Erklärung, die ich dafür hörte, war, als sie die Diagramme entwickelten, von denen sie keine Ahnung hatten, was ein Elektron war. Also haben sie einfach eine Richtung gewählt. Es stellt sich heraus, dass sie es rückwärts hatten und was tatsächlich von + nach - fließt, ist das Fehlen von Elektronen (Löchern).
    Ja, genau das ist damals passiert. Und es gibt im Moment wirklich keinen Grund, die Stromrichtung zu ändern, weil es wirklich keinen Unterschied macht.
    Dies ist kein Missverständnis. In einer Batterie oder Neonlampe fließt die positive Ladung von + nach -. Dann gibt es Halbleiterlöcher. Konventioneller Strom ist genau das: eine Konvention. http://amasci.com/miscon/eleca.html#frkel Forrest Mims verwendet in seinen Einführungsbüchern Elektronenstrom von - bis +. Ich denke, das ist schädlich, aber "keine Seele" ist nicht wahr.
    Pavel Radzivilovsky
    2010-12-01 03:34:24 UTC
    view on stackexchange narkive permalink

    "Langes Haar wächst langsamer" ist auf biologische Effekte zurückzuführen

    Tatsächlich ist dies ein rein mathematisches Phänomen. Je größer die durchschnittliche Länge ist, desto mehr Abnahme wird durch jeden Haarausfall verursacht. Dies führt zu einer Differentialgleichung $$ \ frac {dL} {dt} = K - \ alpha L $$, deren Lösungen exponentiell zu einem Gleichgewicht bei $$ K / \ alpha $$ abfallen.

    Ihr Argument spricht nur über die durchschnittliche Länge der Haare. Oder mit anderen Worten, das ** gesamte Haarvolumen **. In diesem Fall würde es tatsächlich bestimmen, dass eine Person nur ein endliches Haarvolumen erreichen würde, selbst wenn sie unendlich viel Zeit hätte. Dies hat aber nichts mit ** scheinbarer Haarlänge ** zu tun. Dies liegt daran, dass lange Haare leichter zu sehen sind als kurze. Ihr Argument sagt uns also nichts über die Wachstumsrate der scheinbaren Haarlänge aus (was wohl die Leute normalerweise meinen, wenn sie sagen, dass ihre * Haare langsamer wachsen *).
    @marek, ob Wahrnehmung relativ ist oder nicht, ist eine andere Geschichte. Aber auch ohne diese Geschichte ist der interessante Teil genau der folgende: Das Haar muss nicht wissen, wie lang es ist, um länger zu wachsen.
    @Pavel: Ich denke, es ist überhaupt nicht interessant, wenn Sie darauf hinweisen, dass Sie nur von ** Volumen ** sprechen, das unterschiedlich schnell wächst, nicht von der Länge eines einzelnen Haares.
    @marek wie du selbst im Kommentar gesagt hast, gilt dies auch für durchschnittliches individuelles Haar :)
    @Pavel: * individuelles * Haar ist offensichtlich nie * durchschnittlich * und das habe ich nie gesagt. Durchschnitt ist nur eine kollektive Beschreibung einer großen Anzahl von Haaren, und ich behaupte, dass diese Beschreibung für die Frage, warum längeres Haar langsamer wächst, nicht wirklich wichtig ist.
    Dem stimme ich nicht zu. Wenn Sie ein Mädchen sind, das in den Spiegel schaut, können Sie nie ein einziges Haar sehen.
    @Pavel: seufz ... interpretieren Sie meine Aussagen absichtlich falsch? Ich habe nie gesagt, dass einzelne Haare wichtig sind. Nur dass Ihr Durchschnitt keine einzelnen Haare berücksichtigt (insbesondere die langen) und dass der Durchschnitt für diese Frage nicht wichtig ist. In Wirklichkeit ist es wichtig, dass es genügend lange Haare gibt. Das Mädchen konnte unmöglich unterscheiden, ob sie 10000 30-cm-Haare oder 5000 20-cm-Haare und 5000 40-cm-Haare hatte. Ihr Durchschnitt würde gleich ausfallen, aber im zweiten Fall würden ihre Haare offensichtlich 10 cm Songer erscheinen. Ihr Modell berücksichtigt dies überhaupt nicht und ist daher nutzlos.
    Es klingt für mich offensichtlich, dass die Dauer nicht von einer Minderheit der Haare bestimmt wird, sondern von durchschnittlicheren Eigenschaften über eine große Anzahl von Haaren
    @Pavel: sicher, das sage ich. Aber es gibt viele solche Eigenschaften, und Sie haben nur eine verwendet: den Durchschnitt. Und dies ist die gröbste und auch irrelevanteste.
    @there's nichts, was darauf hindeutet, dass es am irrelevantesten ist. Ich denke, es ist in der Tat relevant, am relevantesten.
    @Pavel: Ich habe Ihnen bereits vorgeschlagen, warum es nicht relevant ist. Aber da Sie einfach jeden meiner Einwände ignorieren (ohne ihn jemals zu widerlegen) und offensichtlich denken, dass Sie sich hier möglicherweise nicht irren können, besteht kein Grund für weitere Diskussionen. * (Versuchen Sie übrigens, das nächste Mal @Marek zu verwenden. Es ist schwer herauszufinden, dass Sie etwas anderes kommentiert haben.) *
    @marek Ich habe Ihr Argument noch einmal gelesen und bin dennoch anderer Meinung. Sie würde definitiv 5k und 10k von 40cm Haaren nur durch das Aussehen unterscheiden (selbst wenn es wahr ist, dass das Gesamtvolumen nicht direkt mit der Wahrnehmung zusammenhängt)
    @Pavel: vielleicht würde sie und vielleicht würde sie nicht. Aber der Punkt (dem Sie anscheinend zustimmen) ist, dass die Lautstärke nicht direkt mit der Wahrnehmung verbunden ist. Aber die Wahrnehmung der Länge ist das, worüber die Leute in diesem (angenommenen) Missverständnis sprechen, nicht das Volumen selbst. Das ist also mein Einwand.
    JustJeff
    2010-12-04 21:22:54 UTC
    view on stackexchange narkive permalink

    die Überzeugung, dass die Regeln des Cartoon-Universums für fallende Objekte gelten:

    wie in der Aussage verkörpert: "Wenn Sie in einem fallenden Aufzug gefangen sind, können Sie Zerstörung vermeiden, indem Sie im letzten Moment springen." Wenn der Aufzug fährt, sind Sie in der Luft und fallen nur ein oder zwei Zentimeter. "

    Wenn sich der Aufzug noch nicht zu schnell bewegt (der freie Fall ist also nur für wenige Treppen), hilft das Springen in der Tat **, ** weil Sie Ihre Geschwindigkeit in Bezug auf den Boden verringern (natürlich unter der Annahme, dass es kein Dach gibt im Aufzug). Klar, man muss es schnell machen und auch darauf vorbereitet sein, dass das Springen in 0G nicht ganz trivial ist :-)
    Ich denke darüber nach, dieses B / C zu löschen. Es ist nicht wirklich ein falscher Glaube an die Physik, es ist eher ein physikbezogener urbaner Mythos
    @Marek: Ja, die Leute verstehen den freien Fall nicht und gehen davon aus, dass sie in einer solchen Situation * springen * könnten. Wenn Sie dies tun könnten, würden Sie einen Teil Ihrer potenziellen Anfangsenergie auf den Aufzug selbst übertragen, aber während Sie in Bezug auf den Aufzug * auf * steigen *, würden Sie relativ zum Boden immer noch * ab * gehen. nur etwas weniger als wenn du nicht gesprungen wärst.
    Bei @Jeff: geht es nicht um potenzielle Energie. Sie haben viel Energie in Ihren Muskeln gespeichert und Ihre Beine können als Feder wirken. Wenn Sie wirklich starke Beine hätten (und wieder kein Aufzugsdach annehmen), könnten Sie mit genug Kraft springen, dass es Ihnen gut geht (natürlich unter der Annahme, dass Sie die entsprechende Beschleunigung Ihres Sprunges überleben würden). Übrigens, ich denke, Sie können es hier lassen, es ist ein schönes Beispiel und ich mag unsere Diskussion.
    ok, ich werde es lassen. Ich nehme also an, wenn man in der Lage war, auf eine Höhe H zu springen, dann sollten seine Beine auch in der Lage sein, das PE des Fallens aus der Höhe H zu absorbieren.
    @Jeff: gut, Körper ist nicht vollständig symmetrisch. Normalerweise arbeiten andere Muskeln für eine Richtung als für die andere (medizinischer Jargon sind Agonisten- und Antagonistenmuskeln). Aber abgesehen davon, ja, ich denke schon.
    iii
    2010-12-04 22:24:53 UTC
    view on stackexchange narkive permalink

    Es nervt mich wirklich. Ich habe sogar von einigen Dozenten der Quantenmechanik gehört, dass sie glauben, Quantenverschränkung impliziere schneller als Lichtkommunikation!

    Aber das ist wirklich subtil. Während es sicherlich keine Kommunikation ist, sollte es einen Begriff für "nur etwas weniger als" geben, der Informationen weitergibt. Sie können nützliche Dinge tun, bessere intergalagtische Invasionen planen usw. mit Quantenengantglement, ohne das eine schnellere als leichte Kommunikation erforderlich wäre.
    Eigentlich gibt es einen richtigen Begriff. Es heißt "Nicht-Lokalität".
    Okay, dann lassen Sie es uns so sagen, wenn Sie es vorziehen. Es ist ärgerlich, dass die Leute denken, man könne mit nicht lokalen Merkmalen von Quantenobjekten schneller als mit Licht kommunizieren. (Ich beziehe mich auf keinen Signalisierungssatz)
    Die Quantentheorie ist nicht lokal (und ich bin mir nicht sicher, was Sie mit diesen intergalaktischen Invasionen meinen, @Pavel)! Dies könnte als ein weiteres Missverständnis angesehen werden, das sich aus dem EPR-Paradoxon ergibt. Was EPR (oder besser gesagt Bellsche Ungleichungen) sagen, ist, dass die Quantentheorie ** entweder ** nicht lokal ** oder ** unvollständig ist (im Sinne des Fehlens versteckter Parameter). Wir haben gute Gründe zu der Annahme, dass es lokal ist (z. B. muss QFT der Lokalität gehorchen, wenn dies sinnvoll sein soll). Diese Experimente kamen also zu dem Schluss, dass es keine versteckten Variablen gibt (d. H. QM ist nicht nur Statistik).
    @marek Lookup "Impossible Necklace Game" und andere, Technologie, die Sie mit dieser zweitbesten bis nichtlokalen Kommunikationsfunktion erstellen und verwenden können.
    @Pavel: okay, fertig. Es ist interessant, aber es ist immer noch nur die gute alte Korrelation von verwickelten Drehungen, nichts weiter. Es als nicht lokal (oder als zweitbeste oder was auch immer) zu bezeichnen, ist nur verwirrend und genau der Grund, warum die Leute glauben, dass eine superluminale Kommunikation stattfindet.
    Joe
    2010-12-16 04:41:44 UTC
    view on stackexchange narkive permalink

    Das PI-Team ist die beste Gruppe, um eigene Datensysteme zu entwerfen und zu implementieren, anstatt Personen mit IT-Sicherheit, Datenmodellierung für die Wiederverwendung durch andere Gruppen und anderen Dateninformatikproblemen einzubeziehen.

    Verwandte Missverständnisse: Ihre Daten unterscheiden sich so stark von anderen Daten, dass für jedes neue Experiment ein Datensystem von Grund auf neu entworfen werden muss.

    Gordon
    2011-02-10 11:42:42 UTC
    view on stackexchange narkive permalink

    Das Konzept, dass die Quantenmechanik den Determinismus untergräbt. Die Entwicklung der Schrödinger-Wellengleichung ist vollständig deterministisch. Die Ergebnisse der Messungen sind probabilistisch, dies bedeutet jedoch nicht, dass die verschiedenen überlagerten Zustände keine Ursachen haben. Dies ist nicht dasselbe wie eine Theorie der versteckten Variablen. Die Wahrscheinlichkeiten sind deterministisch. T'Hooft hat einige interessante Ideen zu einem Determinismus, der dem QM zugrunde liegt (nicht dasselbe wie zu sagen, dass die Wellengleichung deterministisch ist). Ich behaupte nicht, dass qm in jeder Hinsicht deterministisch ist, aber es ist auch nicht völlig nicht deterministisch.

    Fakrudeen
    2011-01-28 21:05:25 UTC
    view on stackexchange narkive permalink

    Begriff der Gleichzeitigkeit. Weil die Lichtgeschwindigkeit so groß ist, sieht es in unseren täglichen Angelegenheiten richtig aus.

    Aber es ist wirklich keine existierende Sache [aufgrund der speziellen Relativitätstheorie]. 2 Personen an 2 verschiedenen Orten können nicht "gleichzeitig" sagen.

    adustduke
    2011-01-12 05:02:45 UTC
    view on stackexchange narkive permalink

    Ich finde, die Leute denken, dass "eine Kugel und ein Ball, die aus derselben Höhe geschossen bzw. fallen gelassen werden, nicht gleichzeitig auf den Boden treffen".

    ... weil das eine wahre Aussage ist. Eine Schusskugel startet mit einer deutlich höheren Vertikalgeschwindigkeit als ein fallengelassener Ball.
    Anixx
    2011-01-12 05:31:02 UTC
    view on stackexchange narkive permalink

    Ein häufiges Missverständnis ist, dass die Bewegung massereicher Objekte mit Superluminalgeschwindigkeit in der Allgemeinen Relativitätstheorie verboten ist. Dies gilt nicht, da nur lokale Superluminalbewegungen verboten sind.

    Johannes
    2011-01-14 11:08:56 UTC
    view on stackexchange narkive permalink

    Wie wäre es mit diesen drei Missverständnissen:

    Die Urknalltheorie sagt uns, dass das Universum von einem Punkt aus begonnen hat

    Seit Einstein wissen wir, dass alles relativ ist

    Lichtgeschwindigkeit ist eine grundlegende Konstante

    asmaier
    2011-09-23 15:16:42 UTC
    view on stackexchange narkive permalink

    Die von Albert Einstein 1905 postulierte Konstanz der Lichtgeschwindigkeit wurde durch das Nullergebnis des Michelson-Morley-Experiments motiviert.

    Dies ist falsch. Siehe

    Einstein war hauptsächlich durch die Ergebnisse von Fizeaus Experiment zur Messung der Geschwindigkeit motiviert von Licht in fließendem Wasser

    Ich stimme zu, dass Einstein überhaupt nicht von Michaelson Morley motiviert war. Der Grund, warum die Leute dies sagen, ist, dass Michaelsons Interfermometrie ein grundlegendes Werkzeug ist und die Schüler darauf achten sollten. Aber Einstein war auch nicht wirklich von Fizeau motiviert - obwohl dies ein großes unerklärliches Experiment war, das ihn davon überzeugte, dass das Hinzufügen von Geschwindigkeiten richtig war. Seine größte Motivation von außen war wahrscheinlich Poincare, der 1902 Schwierigkeiten hatte, die Rahmeninvarianz zu verstehen. Er entdeckte fast die Relativitätstheorie, lehnte sie jedoch in letzter Minute ab, weil er die unplausiblen Konsequenzen zu spüren begann.
    Poincaré, du musst ihn wirklich lesen, wenn du es nicht getan hast. Ein bekannter Name, aber nicht so bekannt, wie er sein sollte. Besonders von hochrangigen Leuten.
    Fingolfin
    2011-10-18 10:49:10 UTC
    view on stackexchange narkive permalink

    Dies ist auch ein Missverständnis der Wissenschaft im Allgemeinen, aber ich habe viele Leute sagen hören: "Physik (Wissenschaft) hat bewiesen ..." oder "Können wir Physik (Wissenschaft) verwenden, um dies zu beweisen?" Das Missverständnis ist, dass die wissenschaftliche Methode etwas mit 100% iger Sicherheit beweisen kann. Dies ist sicherlich nicht der Fall; Experimente validieren ein Gesetz nur in Umgebungen, die denen ähnlich sind, in denen die Experimente durchgeführt wurden. Zugegeben, wir können unsere Ergebnisse oft vernünftigerweise weit über bestimmte Einstellungen hinaus verallgemeinern, aber wir können Gesetze nur mit Sicherheit halten, wenn sie getestet und verifiziert wurden.

    jjcale
    2012-05-15 03:05:13 UTC
    view on stackexchange narkive permalink

    Der Treibhauseffekt (nicht der atmosphärische):

    Ein häufiger Irrglaube ist, dass der Grund für diesen Effekt darin besteht, dass Sonnenlicht einfällt und in Infrarotstrahlung umgewandelt wird, die nicht ausgehen kann

    Der Hauptgrund ist jedoch ein mangelnder Luftaustausch (siehe Abschnitt "Echte Gewächshäuser" in http://en.wikipedia.org/wiki/Greenhouse_effect) / p>

    Ich habe dort eine Sekunde gebraucht, aber Sie haben meine Zustimmung. Der Wortlaut ist jedoch ein Auflegen. Sie verwenden den Begriff * Treibhauseffekt * in Bezug auf normale (künstliche) Gewächshäuser. Wir sind so an den Begriff gewöhnt, der in der Klimawissenschaft verwendet wird, dass er mir etwas fremd erscheint.
    Vagelford
    2010-11-19 17:53:15 UTC
    view on stackexchange narkive permalink

    Ein häufiges Missverständnis ist, dass in einem Doppelspaltexperiment die Elektronen oder Photonen gleichzeitig durch beide Schlitze gehen und sich selbst stören.

    Nichts dergleichen passiert jemals und diese Idee stammt nicht aus der Quantenmechanik. Es ist nur bedeutungslos, die Frage (aus welchem ​​Schlitz ist das Teilchen passiert?) Im Kontext des jeweiligen Experiments zu stellen, und noch bedeutungsloser zu sagen, dass es beide durchlaufen hat. Das Partikel passiert immer den einen oder anderen Spalt, wenn Sie ein Experiment einrichten, das die Frage stellt, und wenn Sie die Partikel einzeln abschießen, erhalten Sie immer einen Treffer auf dem Bildschirm und keine Störung von Teilen des Partikels .

    Ich denke, dass das Missverständnis seine Wurzeln in der Wellenanalogie der Wellenfunktionsbeschreibung der Quantenmechanik hat, bei der eine Welle von beiden Schlitzen durchgehen muss, um Interferenzen auf der anderen Seite zu haben. Natürlich bedeutet dieses Bild nicht, dass ein Partikel aus beiden Schlitzen austritt, sondern es gibt nur eine Interferenz zwischen den Wellenfunktionen der beiden unabhängigen Ereignisse an, die sich in einer Verteilung der Wahrscheinlichkeit für das Auffinden eines Partikels an einem bestimmten Punkt auf niederschlägt der Bildschirm. Dies ist eine Aussage von Borns Interpretation, die kürzlich mit einem Dreifachspaltexperiment getestet wurde.

    An dieser Stelle könnte man fragen: „Ok, ich verstehe es für die Elektronen, aber Licht verhält sich im Alltag wie eine Welle. Was passiert in diesem Fall? “Und die Antwort lautet, dass sich Licht nur dann wie eine Welle verhält, wenn Sie einen großen Photonenfluss haben und zur kontinuierlichen Feldnäherung gelangen können. Dann werden die wellenartigen Eigenschaften sowohl für Photonen als auch für Elektronen sichtbar.

    Update: Ein sehr interessantes Video zur Quantenmechanik

    Update2: QM in Ihrem Gesicht

    Update3: Die Feynman-Vorlesungen über Physik Band 3: Streuung von a Kristall (Neutronen).

    ... Sehen wir uns die Physik dieses Experiments an. Wenn wir im Prinzip die alternativen Endzustände unterscheiden könnten (obwohl Sie sich nicht darum kümmern), wird die gesamte Endwahrscheinlichkeit erhalten, indem die Wahrscheinlichkeit für jeden Zustand (nicht die Amplitude) berechnet und dann addiert wird. Wenn Sie die Endzustände auch im Prinzip nicht unterscheiden können, müssen die Wahrscheinlichkeitsamplituden summiert werden, bevor Sie das absolute Quadrat nehmen, um die tatsächliche Wahrscheinlichkeit zu ermitteln. Das, was Sie besonders beachten sollten, ist, dass Sie, wenn Sie versuchen würden, das Neutron allein durch eine Welle darzustellen, die gleiche Verteilung für die Streuung eines sich nach unten drehenden Neutrons erhalten würden wie für ein sich nach oben drehendes Neutron. Man müsste sagen, dass die „Welle“ von all den verschiedenen Atomen kommt und genauso stört wie die hochspinnende mit der gleichen Amplitude. Aber wir wissen, dass dies nicht so funktioniert. Wie wir bereits gesagt haben, müssen wir darauf achten, den Wellen im Weltraum nicht zu viel Realität zuzuschreiben. Sie sind nützlich für bestimmte Probleme, aber nicht für alle .

    Eigentlich ist das, was Sie gesagt haben, nicht wahr. Erstens durchlaufen Elektronen beide Schlitze gleichzeitig (wenn Sie etwas Gegenteiliges sagen wollen, geht es viel mehr um Geschmack oder Interpretation, als um das „Kämpfen mit einem falschen Glauben“). Zweitens, obwohl die Eigenschaften eines einzelnen Photons interessant sind, würde ich nicht sagen, dass es sich nicht wie eine Welle verhält (es hat sicherlich viele wellenartige Eigenschaften).
    Ok, das klingt nach einem "Nein, ist es nicht" -Argument. QM bietet Ihnen einen Rahmen, in dem die Bewegungsgleichungen eines Teilchens (zum Beispiel die Schrödinger-Gleichung) diejenigen sind, die "wellenartige" Eigenschaften haben, und aufgrund dessen erhalten Sie das wellenartige Verhalten von Interferenzen. QM sagt nichts darüber aus, was das Partikel "tatsächlich" tut. Um zu sehen, was das Partikel tut, müssten Sie eine Messung durchführen. Alles andere gehört zum Bereich der „QM-Interpretation“, wo es mehrere Interpretationen gibt und bei einigen einige alberne Dinge vor sich gehen.
    Sehr seltsame Antwort. Tatsächlich hängt es nur von der Interpretation ab, ob Sie glauben, dass das Elektron beide Schlitze passiert. Das bedeutet also, dass es überhaupt kein Missverständnis ist. Tatsächlich sagt Ihnen die attraktivste Interpretation (Feynmans Pfadintegral), dass das Elektron * alle Pfade * durchläuft und alle gleich wichtig für die Bestimmung der endgültigen Amplitude sind. Insbesondere stören die Wege durch linke und rechte Schlitze.
    Es scheint mir, als würden Sie versuchen, alte Vorurteile vorzuschlagen (wie die, die Einstein hatte), dass Quanteneffekte nur als Statistik erscheinen, nachdem Sie * viele * Elektronen durch die Schlitze gelassen haben. Nun, das ist offensichtlich nicht der Fall. Jedes einzelne Elektron verhält sich quantenmechanisch und wandert durch alle Pfade (in der Pfadintegralansicht) oder ist eine Welle, die durch beide Schlitze geht (in der Schrödingerschen Ansicht).
    Ich verstehe nicht, wie Sie zu dem Schluss kommen, was ich schreibe. Die Quanteneffekte sind natürlich von Anfang an vorhanden, aber sie sind in den Bewegungsgleichungen vorhanden. Mein Punkt ist, dass QM nicht angibt, dass "es durch beide Schlitze geht". Ich glaube auch nicht, dass Feynman zustimmen würde, dass der Pfadintegralansatz impliziert, dass ein Partikel allen Pfaden folgt. Es heißt nur, dass jeder Pfad eine Wahrscheinlichkeit hat und ein Partikel tatsächlich nur einem folgt. Am Ende ist es die Wirkung der vielen Partikel, die Ihnen das Interferenzbild geben. Sie können Störungen durch ein Partikel nicht stören.
    @Vagelford: Ich bin mir ziemlich sicher, dass Feynman mir zustimmen würde. Zu meiner Schlussfolgerung haben Sie es einfach selbst gesagt: "Am Ende ist es die Wirkung der vielen Partikel, die Ihnen das Interferenzbild geben." Dies ist im Wesentlichen die alte fehlerhafte statistische Einstein-Sicht auf QM. Wenn Sie annehmen, dass das Teilchen nur einen Spalt durchlaufen hat und die Interferenz nur eine Statistik ist, wäre offensichtlich eine Art Theorie versteckter Parameter erforderlich. Aber wir wissen, dass es solche Ungleichungen nach Bell nicht gibt. Oder vielleicht sagen Sie, dass wir nicht nach dem Weg fragen dürfen. Aber das ist nur eine Interpretation von QM.
    Sie lesen etwas anderes als das, was ich sage. Ich habe nicht gesagt, dass Interferenz Statistik ist und ich spreche nicht über versteckte Variablen. Ich sage, wenn wir nach dem Pfad fragen (ein Experiment einrichten), sehen wir einen Pfad. Wenn wir nicht fragen, dann sagt die Schrödinger-Gleichung nicht, dass sie das durchgemacht hat, das andere oder beides. Das QM-Verhalten liegt in der Dynamik (den Bewegungsgleichungen), nicht in der Kinematik (dem Pfad). Es liegt in der Tatsache, dass die Beschreibung mit einer Wellenfunktion erfolgt, aus der Sie nur Wahrscheinlichkeiten für Ereignisse erhalten, obwohl die Gleichungen vollständig deterministisch sind.
    @Vagelford: okay, ich glaube ich verstehe dich jetzt. Aber Sie diskutieren nur die Interpretation von QM, nicht die eigentliche Physik. Sicher, wenn Sie etwas nicht messen, ist es schwer zu fragen, was passiert (und einige Interpretationen sagen Ihnen sogar, dass Sie * nicht fragen dürfen *). Sie können jedoch eine Interpretation wählen, bei der Sie etwas sagen können, auch wenn Sie nicht messen, solange dies mit den Messungen übereinstimmt. Dies wird als * konsistente Geschichte * Interpretation bezeichnet.
    Ja, aber das ist der Punkt. Es gehört zur Interpretation und nicht zur "eigentlichen Physik" zu sagen, dass es beides durchläuft. Es ist eine schlechte und gefährliche Idee, die beiden zu verwechseln, zumal es keine bevorzugte Interpretation gibt.
    @Vagelford: gut, ja. Aber ich würde das nicht als Missverständnis bezeichnen. Die Idee kann durch korrekte Interpretation richtig gemacht werden. Es spielt also keine Rolle, ob Sie glauben, dass das Elektron durch beide Schlitze geht oder nicht. Es ist nicht ** falsch **. Daher denke ich, dass Ihre Antwort (obwohl sie es verdient, auf die Gefahr hinzuweisen) nicht wirklich hierher gehört.
    Aber es ist ein Missverständnis. Wie würden Sie eine Behauptung nennen, dass etwas im Kontext von QM passiert, wenn dieses etwas nicht ** in ** QM ist?
    @Vagelford: geschieht im Zusammenhang mit einer Interpretation von QM, und wenn die Interpretation selbst konsistent ist, können Sie mit Sicherheit sagen, dass es wirklich im QM selbst passiert ist. Niemand kann dir jemals das Gegenteil beweisen. Das ist aber eher Philosophie.
    Es tut uns leid, abrupt einzugreifen, aber QM sagt uns nichts über den Weg des Elektrons. Das EINZIGE objektive, messbare und nachhaltige, was QM uns sagt, ist das ERGEBNIS der MESSUNGEN. Das heißt, dass Elektronen in einem bestimmten Muster auf den Bildschirm treffen. Der Rest ist (nicht-physische) Interpretation und subjektiv.
    Nun, das ist mein Punkt.
    @Sklivvz:, aber Sie verwenden implizit selbst eine solche Interpretation. Eine, die Ihnen sagt, dass es unphysisch ist, etwas anderes als Messungen zu fragen. Sie haben sich willkürlich dafür entschieden, die Messung zur Grundlage des Physischen zu machen (und scheinen sich dessen nicht einmal bewusst zu sein). Der Punkt ist, ich kann eine andere Interpretation nehmen, die mir sagt, wohin die Elektronen gegangen sind und die auch alle Messungen perfekt reproduziert, so dass es kein Problem damit gibt. Es ist eine ebenso gültige (und physische) Interpretation wie deine ;-)
    @Marek: Ich glaube nicht, dass es eine Interpretation ist, zu sagen, dass QM nicht das eine über das andere sagt. Wenn Sie dagegen sagen, dass das Teilchen beide Schlitze durchläuft, wählen Sie eine oder einige der vielen Interpretationen ohne physikalische Gründe, da es keine bevorzugte Interpretation gibt. Darüber hinaus haben einige der verschiedenen Interpretationen widersprüchliche Ansichten zu einigen Themen. Warum also gegeneinander vorziehen?
    Die Mine @Marek: ist keine Interpretation, sondern lediglich eine Beschreibung dessen, was in den Experimenten aufgezeichnet wird. Daher muss jede Interpretation mit meiner Beschreibung (und den Experimenten) übereinstimmen. Mein Punkt ist, dass unter dieser notwendigen Bedingung alle Interpretationen (vorerst) physikalisch unbeweisbar und als solche völlig subjektiv sind. Wenn Sie keine Interpretation haben, die beweispflichtig ist und durch Experimente tatsächlich widerlegt werden kann, handelt es sich nicht um Physik. Es ist Erkenntnistheorie.
    @Vagelford: Ich denke, es ist eine Interpretation. Sie haben also selbst eine Interpretation gewählt (eine, bei der Sie nicht fragen dürfen). Warum bevorzugen Sie diesen unter allen anderen? Ich sage nicht, dass es ein Problem ist, nur dass es willkürlich ist. Und wie Sie sagen, spielt es für die eigentliche Physik keine Rolle. Ich bin mir also nicht sicher, warum wir das noch diskutieren.
    @Sklivvz: Selbst wenn Sie es leugnen, arbeiten Sie immer noch an einer Interpretation. QM allein ist nur eine mathematische Theorie. Um es mit physikalischer Interpretation zu verbinden, ist es absolut notwendig. Was ist Ihrer Meinung nach die Messung? Denken Sie darüber nach und Sie werden bald feststellen, dass Sie selbst die eine oder andere Interpretation verwenden (wenn auch nur implizit).
    @Marek, Ich weiß / kümmere mich nicht darum, was eine Messung ist, aber ich weiß eindeutig, was das Ergebnis eines Experiments ist. Es könnte sich zum Beispiel um einen fotografischen Film handeln, bei dem einige bestimmte Muster von Partikeln hinterlassen werden. Das muss meiner Meinung nach erklärt werden, und genau das tut QM. QM ist vor allem wegen seiner Terminologie problematisch, da es typischerweise Konzepte aus der klassischen Welt durch Ähnlichkeit entlehnt. Man kann es sich aber einfach als mathematischen Algorithmus vorstellen. Wenn es mehr als das ist, sollte es nur eine einzige gültige Interpretation geben, die durch Experimente bewiesen werden kann.
    @Sklivvz: Ich sehe, es ist dir egal, aber ein Problem zu ignorieren lässt es nicht verschwinden ;-) Wenn du eine vollständige korrekte Beschreibung der Welt willst, musst du deinen Apparat auch durch QM beschreiben. In einigen Fällen ist es nicht möglich, es zu trennen (z. B. in der Quantenkosmologie). Aus diesem Grund stellten die Menschen Fragen zu Messungen und Interpretationen und stellten fest, dass dort nicht triviale Probleme verborgen waren und neue mathematische Theorien (wie Dekohärenz und konsistente Geschichten) geboren wurden. Natürlich können Sie dies für grundlegende Berechnungen ignorieren, aber Sie sollten zumindest wissen, dass Sie unwissend sind ;-)
    @Marek, ernst - Ich verstehe das Messproblem und die unterschiedlichen Interpretationen von QM. Punkt in Frage: Es ist eine Tatsache, dass (soweit ich weiß) jede Interpretation gleichermaßen (in) gültig ist. Mit anderen Worten, die aktuellen Interpretationen sind nicht fälschbar und daher nicht wissenschaftlich.
    @Sklivvz: nicht wahr. Einige Interpretationen sind besser als die anderen in dem Sinne, dass QM besser ist als klassische Mechanik. Z.B. Kopenhagen sagt Ihnen nichts über die Messung, aber andere Interpretationen (und sie sagen Ihnen auch, warum Kopenhagen in den meisten Fällen funktioniert). Und zu sagen, dass Interpretationen nicht wissenschaftlich sind, ist einfach falsch. Sie können sicherlich eine Interpretation des QM vorschlagen, die nicht mit den Beobachtungen übereinstimmt, und eine solche Interpretation würde verfälscht. Wie auch immer, Sie ** brauchen ** eine Interpretation, um QM zu verwenden (auch wenn es nur Ihre ist * halten Sie den Mund und berechnen Sie * Ansatz).
    @Marek, Ich sehe, dass Sie Argumente genauso mögen wie ich. Ich glaube, wir sind uns nur nicht einig darüber, was eine Interpretation ausmacht. Ich denke nicht, dass meine eine Interpretation ist, aber Sie tun es. Mit meiner Definition habe ich recht und mit deiner hast du recht. Als solches stimmen wir zu, nicht zuzustimmen :-)
    @Sklivvz, Nun, ich mag keine Argumente, aber aus irgendeinem Grund neige ich immer dazu, in sie verwickelt zu werden :-) Okay, wir können uns darauf einigen, nicht zuzustimmen. Aber nur dass Sie wissen, dies ist [meine Definition der Interpretation von QM] (http://en.wikipedia.org/wiki/Interpretations_of_quantum_mechanics) und es ist ziemlich Standard. Wenn Sie über etwas anderes sprechen, sollten Sie die Leute wissen lassen und sie nicht mit nicht standardmäßigen Begriffen verwechseln ;-)
    @Marek, Ich habe mich auf dasselbe bezogen ... aber ich möchte das Argument nicht neu starten (<-intended in einem guten Sinne) :-)
    @Sklivvz, in Ordnung, lass es sein :-) Deshalb denke ich, dass Vagelfords Antwort nicht gut ist. Es geht nur um Interpretation (der Interpretation) :-)
    @Marek, Ich stimme zu.
    Ich sollte sagen, dass ich auch denke, dass Interpretation das ist, was im Wikipedia-Artikel beschrieben wird. Ich denke, meine Antwort unterscheidet genau zwischen QM und Interpretation und weist im Wesentlichen darauf hin, dass es im QM-Unterricht einen bevorzugten Stil gibt, der eine oder mehrere Interpretationen fördert. Ich finde keinen Grund, warum das passieren sollte und ich denke, ich bin nicht der einzige, wie man in den Videos sehen kann.
    Lehren oder Kommunizieren der Quantenmechanik im Allgemeinen.
    @Vagelford: Ich stimme zu, dass die beiden klar getrennt werden sollten. QM kann allein als mathematische Theorie ausführlich allein studiert werden. Dies wird jedoch normalerweise nicht gelehrt. Kopenhagen wird normalerweise angenommen und die Menschen müssen sich oft über die Art der Messung und andere Zusammenhänge mit der Physik wundern, ohne jemals über Interpretationen informiert zu werden.
    Marek, da sind wir uns offensichtlich einig.
    @Vagelford, also gut. Übrigens, wenn Sie die Frage wirklich so formuliert haben, dass klar ist, dass Sie über die Unterscheidung zwischen reinem QM und Interpretationen sprechen, würde ich Ihnen eine Gegenstimme geben. Aber so wie es aussieht, ist es wirklich verwirrend (zumindest für mich; und ich nehme an, es muss noch mehr für jemanden sein, der nichts über Interpretationen weiß).
    dan_waterworth
    2010-12-15 13:19:43 UTC
    view on stackexchange narkive permalink

    Hier ist ein falscher Glaube. Alles, was Ihr Physiklehrer sagt, ist wahr. Weil Physiklehrer Sie niemals darüber irreführen würden, wie viele Materiezustände es gibt oder wie Auftrieb auf einem Flügel erzeugt wird.

    Erich Wang
    2011-01-14 03:52:41 UTC
    view on stackexchange narkive permalink

    Wie wäre es damit - diese Parität ist eine der grundlegendsten Symmetrien (was nicht der Fall ist).

    Ich kenne die Experimente, die eine Paritätsverletzung zeigen. Ich denke jedoch, was ich immer noch nicht verstehe, ist Folgendes: Rotationssymmetrie ist grundlegend. Und die Parität dreht sich nur um 180 Grad. Wenn wir also sagen, dass jede Rotationssymmetrie fundametal ist, warum sollte eine Teilmenge (die sich mit einem ganz bestimmten Winkel dreht - 180 Grad) nicht grundlegend sein? Widerspricht das nicht dem Selbst?

    Das ist nur in zwei Dimensionen der Fall. Im 3D-Raum entspricht eine Paritätstransformation keiner Drehung.
    @David: in 2d-Parität wird definiert, indem 1 Achse reflektiert wird, nicht beide, die in jeder geraden Dimension gleich sind.
    @Ron: stimmt, ich glaube, ich habe einen Fehler gemacht.
    Diego
    2011-02-27 10:16:45 UTC
    view on stackexchange narkive permalink

    Ich empfehle dringend, den Artikel "Quantenmechanik: Mythen und Fakten" von H. Nikolic zu lesen http://arxiv.org/abs/quant-ph/0609163

    Einige Themen umfassen Welle-Teilchen-Dualität, Zeit-Energie-Unsicherheitsrelation und fundamentale Zufälligkeit.

    Ich habe diesen Artikel in anderen Communities diskutiert und er scheint zuverlässig zu sein.

    AdamRedwine
    2011-08-25 17:28:05 UTC
    view on stackexchange narkive permalink

    Missverständnis - Das Unsicherheitsprinzip ist eine Aussage über "unsere" Fähigkeit, Messungen durchzuführen.

    Korrektur - Das Unsicherheitsprinzip ist ein Ergebnis der Natur der Partikel selbst und bezieht sich auf die Fähigkeit von irgendetwas , "die relevanten Messungen durchzuführen". Es sind nicht nur wir, die die gleichzeitigen Werte inkompatibler Observablen nicht bestimmen können, Gott kann es auch nicht.

    Der Punkt ist, dass wenn wir es nicht messen können, es nicht existiert. Die beiden sind Synonyme. Dies ist eine wichtige philosophische Position.
    Ich denke, es ist wichtig, das "Wir" aus der Aussage herauszunehmen. "Wenn es nicht gemessen werden kann, existiert es nicht" ist näher am Punkt.
    ja, du hast recht.
    Siyuan Ren
    2011-08-26 13:36:24 UTC
    view on stackexchange narkive permalink

    Die Erde dreht sich um die Sonne. Es ist falsch zu sagen, dass sich die Sonne um die Erde dreht.

    FAKT: Bewegung ist relativ. Es ist nichts Falsches daran zu sagen, dass sich die Sonne um die Erde dreht. Ersteres wird häufiger erwähnt, weil die Sonne (oder genauer gesagt das Schwerpunktzentrum) ein besserer Trägheitsrahmen ist und andere Planeten sich ebenfalls in einem nahen Kreis um die Sonne drehen, aber auf bizarre Weise um die Erde.

    Wenn man sich im Vakuum schneller als Licht bewegt, kann man in die Vergangenheit zurückkehren.

    FAKT: Keine Erklärung erforderlich.

    Im erdzentrierten Rahmen fällt das Licht von entfernten Sternen auch einmal im Jahr um die Erde herum ab, weil alle entfernten Sterne und das Licht ebenfalls wackeln. Es ist ein Unsinnsrahmen für die Kosmologie, aber im Prinzip und im Prinzip funktioniert es nur.
    kiss my armpit
    2012-06-27 11:10:19 UTC
    view on stackexchange narkive permalink

    Das Missverständnis: Naive Menschen gehen davon aus, dass ein Objekt schwebt, wenn es unabhängig von der Schwerkraft in einen Vakuumbehälter gestellt wird.

    Die richtige Tatsache: Das Fehlen der Schwerkraft lässt das Objekt schweben.

    James Cooper
    2012-09-01 09:19:22 UTC
    view on stackexchange narkive permalink

    Es ist eine verbreitete falsche Überzeugung unter zynischen Physikern, dass es keine physikalische Bedeutung hat, zu fragen, "warum es etwas gibt, anstatt nichts". Die Frage, ob es ein unvermeidliches, selbstkonsistentes, selbstreferenzielles mathematisches Gesetz gibt, das die Existenz des Universums vorschreibt, ist real und legitim. Eine einheitliche Theorie wäre ein Schritt in die richtige Richtung. Hierbei spielt auch die Forschung in Zahlentheorie, Primzahlen, Unendlichkeit usw. eine Rolle.

    Marty Green
    2011-06-08 09:42:42 UTC
    view on stackexchange narkive permalink

    Ich weiß, dass es eine alte Frage ist, aber es macht zu viel Spaß, diese Frage weiterzugeben. Es gibt einige Beispiele für falsche Ideen, die sich durchgesetzt haben, aber für die Zentralität der Physik und den Grad, in dem sie falsch und irreführend ist, ist es schwierig, ein besseres Beispiel als das Bohr-Atom zu finden. Als es herauskam, elektrisierte es die Welt der Physik aufgrund seiner revolutionären Sichtweise und seines erstaunlichen Erfolgs. Aber elf Jahre später wurde es völlig veraltet. Trotzdem erregte es in dieser kurzen Zeit die Vorstellungskraft sowohl der Physikwelt als auch der Öffentlichkeit bis zu dem Punkt, an dem es bis heute das ikonische Bild des Atoms in den Köpfen nahezu aller bleibt.

    Die irreführenden Einflüsse dieses Modells sind äußerst weitreichend. Man kann mit der Idee des "Quantensprungs" beginnen, der den Zusammenbruch der Wellenfunktion hervorrief und mehrere Universen usw. hervorbrachte. Aber das vielleicht beständigste Überbleibsel des Bohr-Atoms ist die Vorstellung, dass die gewöhnlichen Gesetze der Elektromagnetik auf atomarer Ebene aufgehoben werden müssen, sonst wäre das Atom instabil. Natürlich trifft in der modernen Quantenmechanik nichts dergleichen zu, am allerwenigsten für das Wasserstoffatom. Der größte Triumph der Schrödinger-Gleichung bestand darin, zu zeigen, dass die Bewegung der elektrischen Ladung über die Zeit verfolgt werden kann und dass die stabilen Atomkonfigurationen genau diejenigen ohne beschleunigende Ladungsverteilungen sind. Dies war eine unmittelbare und offensichtliche Folge von Schrödingers Lösung für das Wasserstoffatom

    Ich würde behaupten, dass es nicht nur wahr ist, dass die stabilen Konfigurationen im QM solche ohne beschleunigende Ladungen sind. Ich würde noch weiter gehen und vorschlagen, dass in den Fällen, in denen sich die Ladungen beschleunigen, die Geschwindigkeit, mit der die kinetische Bewegungsenergie in elektromagnetische Energie umgewandelt wird, genau mit der übereinstimmt, die auf gewöhnliche Weise unter Anwendung der Maxwellschen Gleichungen berechnet würde. Wenn man beispielsweise bei einem Schwarzkörperstrahler einfach die vibrierenden Atome nimmt, die Beschleunigungsbewegung der Ladungen berücksichtigt und die klassische Antennentheorie anwendet, erhält man das richtige Schwarzkörperspektrum. P. >

    -1: Nein, das ist nicht korrekt. Das Bohr-Modell mit seinen Quantensprüngen ist genauer als die Schrödinger-Beschreibung von 1926, da die Wellenfunktion kein klassisches Feld sein kann, weil sie für zwei Elektronen in 6 Dimensionen vorliegt. Bei mehr Partikeln kommt es zu vielen dekohärenten Welten und zum Zusammenbruch der Wellenfunktion, und die Ladungen beschleunigen sich in allen Lösungen, einschließlich des Grundzustands von Wasserstoff.
    Es fällt mir schwer zu verstehen, was Sie mir sagen wollen. In welcher Interpretation beschleunigen sich die Ladungen im Grundzustand von Wasserstoff? Mehr auf den Punkt, wenn Sie sagen "das ist nicht richtig", frage ich mich, ob Sie einen bestimmten Fehler in dem, was ich gesagt habe, identifizieren können?
    Die Beschleunigung eines Quantenteilchens wird durch den Operator $ dp \ über dt $ angegeben, der nur dann Null ist, wenn sich das Teilchen in einer ebenen Welle befindet. Der Wasserstoffgrundzustand beschleunigt sich. Alles, was Sie gesagt haben, ist falsch, daher ist es schwierig, einen Fehler zu finden.
    Ach komm schon. Im Grundzustand des Wasserstoffatoms gibt es keine beschleunigende Ladung, und das weiß jeder.
    Jeder, der weiß, weiß das Gegenteil.
    Sklivvz
    2010-12-15 05:56:45 UTC
    view on stackexchange narkive permalink

    Das ist ein Augenzwinkern!

    Dass ein Entleerungstank mit einer nach unten zeigenden Düse tatsächlich eine Kraft erfahren würde!

    (siehe http: // arxiv .org / pdf / physics / 0312087v3)

    :-)

    Lagerbaer
    2010-12-01 07:17:06 UTC
    view on stackexchange narkive permalink

    Etwas kann nicht aus dem Nichts kommen

    Ja, es kann: In Quantenfeldtheorien ist das Vakuum nicht leer .

    In QFT ist Vakuum sicherlich nicht ** nichts ** (es ist zufällig ein Zustandsvektor, der von einigen Operatoren vernichtet wird), daher hat dies nichts mit Ihrem vorgeschlagenen Missverständnis zu tun.
    Sehen Sie "Universe from Nothing" von Lawrence Krauss. http://www.youtube.com/watch?v=7ImvlS8PLIo
    Ich habe keine Zeit, es mir anzusehen. Wenn es eine Pointe hat, dann sag es mir einfach. Wie auch immer, * nichts * hat entweder in einer Theorie einen präzisen mathematischen Sinn (und dann ist es sicherlich nicht nichts) oder es ist nur ein philosophisches Konzept (und daher keine Physik). In jedem Fall ist Ihre Antwort nicht gut.


    Diese Fragen und Antworten wurden automatisch aus der englischen Sprache übersetzt.Der ursprüngliche Inhalt ist auf stackexchange verfügbar. Wir danken ihm für die cc by-sa 2.0-Lizenz, unter der er vertrieben wird.
    Loading...